搜索内容

您当前的位置:首页 > 办公技巧 > 正文

七年级英语试题 七年级英语试题分析

办公技巧
日期·2024-10-10 09:08

2017年七年级英语下册期末试卷及

“__9__, your father doesn’t give the money for the meal and your -cream,” the waiter stopped him and said.

七年级英语 期末马上来了,愿你们考出好成绩,下面是我为大家精心整理的2017年七年级英语下册期末试卷,仅供参考。

七年级英语试题 七年级英语试题分析七年级英语试题 七年级英语试题分析


2017年七年级英语下册期末试题

(总分:120分,时间:120分钟)

卷一(75分)

听力部分(20分)

A. 听 句子 ,选择恰当的答语,每题读两遍(每小题1分,计10分)

1.A.It’s interesting B.He’s in the library C.A doctor.

2.A.Shelikes doing housework B.She is a teacher

C.She likes teaching.

3.A.He is very tall B. He is a student C.He is from America

4.A.Yes I was B.Yes I did C.Yes I am

5.A.I’m sorry to hear that. B.Thank you very much

C.You’re welcome.

6.A. She is at home B.She is twelve C.she is fine

7.A.It’s a hospital B.It’s next to the t off C.It’s one the street.

8.A.I’d like a pizza,please B.Yes,do please C.What’s it

9.A.It’s five yuan B.It’s large C.It’s very delicious

10.A.Yes, you can B.Who are you? C.Yes speaking

B听对话,选择所听问题的正确,每段念两遍(每小题1分,计5分)

11.A.He is short with cy hair B. He is tall with cy hair

C.He is tall without hair

12.A.A cup of milk B.A cup of cof fee C.A cup of tea

13. A.Soccer B. Tennis C. Basetball

14.A.Under the desk B. Near the door C. Behind the door.

15.A.Japanese B. Chinese C. English

C听短文,根据短文内容,选出一个正确回答所提的问题,短文读三遍。(每小题1分,计5分)

16.What is Mr King?

A.A Chinese worker B. An English teacher

C. A Chinese teacher.

17.Where is he from?

A. Australia B. Austria C. America

18.How many children does he he?

A. Only one B. Two C. Three

19.Where do his children go the school?

A.In Japan B.In the USA C. In China

20.What do they often do after school?

A. Shopping and fishing

B. Swimming and walking C.Swimming and shopping

A. 选择填空(共30小题,每小题0.5分,计15分)

A.词语释义

21.I’m free today.

A. busy B. not busy C. no busy D. busier

22.Did Jimmy he eggs for lunch?

A. get B. has C. had D.eat

23.I he no idea.

A. doesn’t know B. not know C. don’t know D. know

24.He fun in the park

A. Enjoy oneself B.Enjoy yourself

C.He a time D.Enjoy you

25.He does nothing on Sunday.

A.Doesn’t do anything B.doesn’t anything

C. does do nothing D.does do anything

B、单项选择

26.The book store is far away from here. You’d better____________.

A.by bus B.on a bus C. take a bus D. in a bus

27.-Would you please go out for a walk with me?

-_______, but I’m too busy now.

A. That’s right. B.I’d love to

C. Of course not D.That’s all right

28.-I’m gry ,Mom. --__________

A .What about a pizza? B.I’m gry ,too

C. It’s time for lunch D.What about you?

29._________interesting dolphin it is!

A. How an B.How a

C. What an D.What a

30.Thank you for_________me the good news

A. l B. to l C. ling D.told

31.Chinese people’s forite food is ________

A. r B. hamberger

C. hot dog D. lemonnada

32.--________do you want to be an actor? --It’s very interesting.

A. What B.When C. How D.Why

33.--Would you like some drink? --_______.

A. Here you are B. Yes, just a little

C.Please give me some D.Yes’ I like

34.If you are gry ,you can buy_______in a supermarket.

A. Water B. house C. drink D. food

35.Is there a pay phone________the neighborhood?

A. in B. on C. to D. off

36.I________know where the t off is.

A. not B. don’t C.doesn’t D.am not

37.Why_______he like koala bears?

A. is B. are C. do D. does

38.________late for school again.

A. Not B.Not be C.Don’t be D. Aren’t

39.--___________pizza would you like? --A all one ,please

A. What kind of B.What size C.How many D.How much

40.--________you’re your uncle do? – He is an engineer.

A. What B.Where C. How D.Why

41.There__________some d tea in the cup

A. are B.is C.he D.has

42.Whose book are these? – They are________.

A. him B. her C.Mr. Green D.Mr. Green’S

43.--_______did you go last Sunday? -- I went to the zoo

A. How B.Where C.When D.Why.

44—Do you like the pop music? -- No, I don’t . They’re__________.

A.silly B.pretty well C. beautiful D. awful

45Go straight and ___________left. The hospital is next to the t off.

A. turn B.take C.go D. carry

46.--What________your English teacher_____like?—She is tall with long hair.

A.is, look B.is, looks C.does, look D.do, looks

47. The tall boy is good at_________football.

A. play B.plays C.played D.playing

48.-What is John like? -________

A.He is tall and handsome B.He likes reading books C.Shy D.Baby face

49.My friend ________a medium build and she ________medium height.

A.has, has B.is , is C.has , is D. is , has

50.--How was your vacation, Sarah? --________.

A. It’s pretty good B.It was pretty good

C.It’s hot D.It was hot

五、完形填空(10分)

It 51 Sunday yesterday . We has 52 classes. I got up at 6:30 53 morning. Then my mother and I went to the shop. We wanted to do some 54 . My mother wanted to buy some food 55 super and I wanted to buy some school things: some books and a pen. The shop 56 early in the morning. 57 a lot of people in the shop. They were 58 , old and young. Some things in the shop were cheap ,some were not. We bought some food and school things. The people in the shop 59 friendly 60.

51.A.is B.was C.it’s D.it was

52.A.no B.not C.not any D.much

53.A.in B.in the C.at D.at the

54.A.shops B.shoping C.shopping D.shop

55.A.for B.to C.with D.at

56.A.open B.opening C.is opening D.opened

57.A. There is B.There has C.There were D.There he

58.A.man and woman B.mans and womans C. men and women D.a man and a woman

59.A.was B.were C.did D.he

60.A.for we B. to us C. for our D. to ours

六、阅读理解:(共15小题,每小题1分,计15分)

ABilly and Sam were twins. They were in the same class. They were both very happy. One day their teacher asked the class to write a comition( 作文 )“My Mother”. Sam wrote one, but Billy was lazy. He just copied his brother’s. The next day, the teacher asked Billy why his comition was the same as Sam’s. “We he the same mother, don’t we?”answered Billy.

根据短文内容判断句子正误,正确的填A,错误的填B。

61.Sam was older than Billy.

62.Sam and Billy were in different classes

63. The name of the comition was”My Mother”

64.Billy’s comition was the same as Sam’s

BTV PROGRAMMERS(节目)

Channel 1 Channel 2

18:00 Around China 17:45 Comrs today

18:30 Cartoon network 18:10 Chinese arts

19:00 News 18:30 English classroom

19:30 Weather report 18:50 Animal world

19:40 Around the world 19:25 China’s 2004

20:10 TV play: Sisters 20:20 Sports

21:00 English for today 21:00 TV play: Big family

21:15 Popular music 21:45 English news

21:55Talk show 22:00 Music

66.You want to know soming about Japan, you can watch________

A.Weather report B.China’s 2004 C.Around the world D.Animal world

67.You are a football fan. You may watch TV at___________.

A.19:00 on Channel 2 B.19:40Channel 1 C.20:20Channel 2 D.21:15Channel

68.How long does the TV play“sisters”last(持续)?

A.30 minutes B.50 minutes C.90 minutes D.120 minutes

69.You can watch _______if you want to learn English.

A.English news B.English classroom C.English for today D.A、B and C

70.Children often like to wat ch_________.

A.Cartoon network B.News C.Weather report D.Sports

CTwo men are walking in the park. One is Mr Brown , a worker, the other is Mr Black. Mr Brown is going to a chair. It is beside the road. Mr Black is after him. Mr Brown finds Mr Black. Now Mr Black begins to walk faster(更快). Because he wants to get to the chair before Mr Black. Now Mr Black begins to walk faster because he also wants to get to the chair first. Then they both run .Mr Black runs faster than (比) Mr Brown. So he gets to the chair forst and sits down on it. Mr Brown runs after Mr Black. When he gets to the chair. He sees Mr Black sitting on it. He is sorry to show a not(通知)with the words“Wet Paint(油漆未干)”to Mr Black . Now Mr Black knows why Mr Brown is going to the chair.

71. Where are Mr Brown and Mr Black?

A.In the garden B. On the chair C. In the park D.Beside the road

72.What are they doing? They are________

A. talking B.walking to the chair C.holding a not D.sitting on a chair

73._________is after_________at first

A.Mr Black ,Mr Brown B.The worker, Mr Brown C.Mr Brown ,Mr Black D.Mr Brown The worker

74.________get to chair first.

A.The worker B.Mr Brown C.Mr Black D.A strange(陌生人)

75.Mr Brown goes to the cC.Yes, I've D.Hehair because__________.

A.he wants to sit there B.it is his chair

C.he wants to talk to Mr Black on the chair D.he wants to l people ”Wet Paint”

七、交际用语。

A.根据对话内容,从方框各选项中选出能填入空白处的选项。(每小题1分,计5分)

A.That’s a good idea

B. I’m calling to thank you for your present.

C. Yes, very much.

D.Well, I saw many old things on show.

E.I called you yesterday, but you were not in.

A:Hello, Jim .This is ANN speaking.

B:Hi, Ann.

A: 76

B:Do you like it?

A: 77 It’s a beautiful sweater. I really like the color.

B:I think the blue is just the right color for you . By the way, you looked wonderful at the party.

A:Thank you ,I really enjoyed it. 78

B:Oh, I’m sorry. I went to the Science Museum.

A:What did you see?

B: 79

A: Wow! Are there any new things ?

B: Yes, many more! Why not go there and he a look?

A:Don’t worry. I’ll go with you.

B: 80 But I don’t know the way to the museum.

A:Don’t worry. I’ll go with you.

B:Great! Thanks.

B、从Ⅱ栏中找出Ⅰ栏中问句的适当答语。(共10小题,每小题1分,计10分)

Ⅰ Ⅱ

81.What does your brother look like? A. Yes, they did.

82.How was your weekend? B. It’s ten yuan

83. What time did Tom get to school? C. He’s tall with brown hair

84.Did your parents go to the movies yesterday? D. At about six o’clock

85.Where does Mr Smith come from? E. Yes, there is.

86. What is your best friend like? F. They are playing soccer.

87.Are the children playing soccer or basketball? G. I like Chinese.

88. Do you like English or Chinese? H. America.

89.How much is the book? I. S he is really friendly.

90.Is there a t off near here? J. It was great.

2017年七年级英语下册期末试卷参

题号 1 2 3 4 5 6 7 8 9 10 11 12 13 14 15 16 17 18

B B A A C C B A B C B A C C B B A B

题号 19 20 21 22 23 24 25 26 27 28 29 30 31 32 33 34 35 36

C C B D C B A C B A C C A D B D A B

题号 37 38 39 40 41 42 43 44 45 46 47 48 49 50 51 52 53 54

D C B A B D B D A C D C C B B A[ B B

题号 55 56 57 58 59 60 61 62 63 64 65 66 67 68 69 70 71 72

题号 73 74 75 76 77 78 79 80 81 82 83 84 85 86 87 88 89 90

A D D B C E D A C J D A H I F G B E

一、1. She went to the zoo She saw the monkeys

2.He went to the park he played soccer.

3. No, he didn’t Yes , he did

二. 1.eating 2.was 3.speak 4.Don’t open 5. likes

6. saw 7.are 8. take 9.to play 10. did

三、1.got to 2.What day 3. Did go 4. Does he 5. I didn’t

6. How beautiful 7. What forite 8. When did 9. was yesterday 10.are playing

2017年七年级英语下册期末试卷及相关 文章 :

1. 2017年七年级下册英语期末试卷

2. 人教版七年级英语下册期末试卷及2017

3. 2017年七年级期末英语试卷

4. 七年级下册英语期末卷及2017

5. 2017新目标七年级下册英语期末试卷及

七年级上册期末英语试卷(人教版)

49. speak to50. he to

人教版七年级英语上册期末试题(含)

楼德镇中学试题库 2010-10-04 11:29:59 阅读378 评论2 字号:大中小

七年级英语期末试题

卷选择题(50分)

I.单项选择。(每小题1分,计15分)

( )1. This is _________ alarm clock.

A. a B. an C. the D. /

( )2. We don’t like_________.

A. broccoli B. broccolis C. a broccoli D. broccolies

( )3. _________. Is this your pen?

A. Sorry B. Hello C. Excuse me D. Hi

( )4. Do you he two ________?

A. tennis racket B. tennis rackets

C. tennis’ racket D. tennis’ rackets

( )5. He _________ his homework at school.

A. doesn’t do B. don’t does

C. isn’t do D. does not

( )6. ________this your guitar?

A. Am B. Is C. Are D. Do

( ) 7. Let’s _________ now.

A. go to home B. to go home C. go home D. to go to ( ) 8. How much are these shoes? _________ twenty dollars.

A. Are B. There C. They’re D. Their

( ) 9. My father usually________ a shower _________the morning.

A. take; in B. takes; in C. take; on D. takes; at

( )10. ________you play the violin?

A. Are B. Can C. Is D. When

( )11 ---This is my sister Ann.

---Is _____ a student?

A. she B. he C. her D. his

( )12. I ________ two baseballs and my friend ________five baseballs.

A. he; he B. he; has C. has; he D. has; has

( ) 13. ---Is that your book?

---Yes, ________.

A. it is B. it isn’t C. it’s D. this is

( )14. Miss Wei often s us ________ our English.

A. at B. in C. of D. with

( )15. I want __________ the music club.

A. join B. to join C. joins D. be join

II.从A、B、C、D中选择最恰当的选项填进文章中去,你会读到一个有意思的故

事。(15分)

I like my Chinese teacher very much. He is a 16 man, about 25 years old. He is tall 17 black short hair. He is a very humorous(幽默的) person. He often 18 us jokes(笑话) and 19 to make our class more 20 . In class, he is a very 21 teacher. If we don’t listen to him carefully, he will give us some punishment(惩罚) by 22 us some questions. But after class, he becomes a very good 23 of us. He often talks with the girls and plays 24 with the boys. All of us really like him. He is our 25 teacher.

( )16. A. old B. young C. very old D. aged

( )17. A. and B. for C. with D. at

( )18. A. ls B. says C. speaks D. speak

( )19. A. a story B. storys C. story D. stories

( )20 A. tired B. difficult C. interesting D. boring

( )21. A. busy B. strict C. interested D. well

home

( )22. A. ask B. asks C. asking D. asked

( )23. A. teacher B. teachers C. friend D. friends

( )24. A. a basketball B. the basketball C. basketballs D. basketball

( )25. A. forite B. the forite C. foritest D. a forite

III.经过一学期的英语学习,你的阅读理解能力一定有了很大提高。(20分)

ADear Mr. Wang,

Someone comes to the off to see you in the morning, but you and your wife are out. He comes here at 10am. Because he is very busy, he goes away half an hour later. He ls me that he is your clasate at college(大学). Now he teaches Chinese in a high school(高中) in this city and he lives near the No. 6 Middle School. He is a very tall man with short hair. He wears a pair of glasses. He ls me his ephone number is 33426685. He is often at home at 7:00 pm. So you can call him in the ning.

( )26. The author(作者) writes this note(便条) to ______.

A. Mr. Wang B. Mr. Wang’s wife C. Mr. Wang’s clasate D. me

( )27. Mr. Wang’s clasate is __________.

A. a teacher B. a worker C. a farmer D. an actor

( )28. When is Mr. Wang’s clasate often at home?

A. In the morning B. In the afternoon. C. At 7:00pm. D. We don’t know.

( )29. How long does Mr. Wang’s clasate stay in the off?

A. Ten hours. B. An hour. C. Two hours. D. Half an hour.

( )30. Which of the following is TRUE?

A. Mr. Wang and his wife are clasates.

B. Mr. Wang’s clasate is short.

C. Mr. Wang goes out with Mrs. Wang in the morning.

D. Mr. Wang’s clasate meets Mr. Wang in the morning.

BI will nr forget my first English class. It is very interesting. It’s the first class in the morning. The teacher goes into the classroom with a ile on his face. At the beginning, he makes a self-introduction(自我介绍). And he ls us that his English name is John. Then he asks all of us to think of English names for ourselves. We are very excited(兴奋的) to find a good name. Then he asks us to write our names down on a piece of . I think about my English name for a long time. Then I come up with(想出) a beautiful name ---Shirley. All the students write their names down and give the to the teacher. When the teacher calls “Shirley” to answer his question, three girls stand up at the same time.

( )31. What do we do in the first English class?

A. Tell the teacher our Chinese names. B. Find English names for ourselves.

C. See an English film. D. Draw pictures.

( )32. What does our teacher do first?

A. He asks ry student to make a self-introduction.

B. He introduces(介绍) himself to all of us.

C. He asks us many difficult questions

D. He ls us a story.

( )33. What’s the English name of our English teacher?

A. Jack. B. Shirley C. John. D. Tom.

( )34. Where do we write down our English names?

A. On the blackboard. III. 1-5 DBCBA 6-10 ACBBC B. On the desk.

C. On our hands. D. On the .

( )35. Which of the following is TRUE?

A. The teacher gives ry student an English name.

B. Our teacher is a young girl.

C. Three girls choose(选择)the name “Shirley”.

D. The teacher calls our Chinese names in class.

IV 单词拼写(每小题1分,共10分)。

36. My mother and father are my p______________.

37. I’m a student. I do my h______________ ry day.

38. C____________ is our homeland(祖国).

39. E____________ me, is this your pen?

40. Bill’s pants are long, but Emma’s pants are s____________.

V选词填短文(15分)

song paint clasate .play Saturday sing friend music swim same different picture

I he four good __41____. They’re Jennifer, Victor, Cindy and Leila. We are in the _42____ class. So we’re _43______ too. Jennifer swims very well, she wants to join the ___44____ club. Then she can swim on ___45____or Sundays. Victor wants to join the art club because he is good at __46______. Cindy is a pop music fan. She can sing a lot of __47___. So she wants to join the __48____ club. Leila wants to join the music club because she __49____ the violin very well. And she wants to be a ___50____ when she grows up(长大). I like them.

VI任务性阅读(10分)

My name is Tommy Wang. T-O-M-M-Y, Tommy, W-A-N-G, Wang. I am a Chinese boy. My ephone number is 85523963. My ephone is white. Look! This is my pencil box. ① It’s big and beautiful. It’s white. What’s in it? This is ② ________ eraser. It’s white. This is a pencil. It’s white. ③ This is a ruler. It’s white, too. I like white. I like green, too. Look! ④ My schoolbag is green. My notebook is green, too. But my forite color is white.

This is my good friend. His name is John. ⑤ He is English. Linda is John’s sister. I like my friend. Like English.

51. 把第①处黑体字部分的句子译成汉语。

___________________________________________________________

52. 在第②处填入一个适当的冠词。

___________________________________________________________

53. 把第③处黑体字部分的句子改成复数。

___________________________________________________________

54. 就第④处黑体字部分句子的划线部分提问。

___________________________________________________________

55. 将第⑤处黑体字部分的句子改为同义句。

___________________________________________________________

VI.书面表达(共15分)

请根据下列提示,写一篇10句话(大约50-70个字左右)的自我介绍。

(1)我叫Frank, 今年14岁,是一名Guang Ming Middle School的学生;

(2)可参照以下句型:I can ….

I like ….

My forite… is/are ….

My hobby is …/ My hobbies are ….

I go to school ….

七年级英语参

笔试部分

I 1—15 BACBA BCCBB ABADB

II 16—25 BCADC BCCDA

III A: 26—30 AACDC

B: 31—35 BBCDC

IV36parents37homework38China39Excuse40short

V41friends42same43clasates44swimming45Saturdays46painting47songs

48singing49plays50musician

51. 它大又漂亮。 52. an 53. These are rulers.

54. What color is your schoolbag? 55. He comes from England.

七年级英语下册次月卷(2)

二、完形填空 (10分)

Li Lei gets an e-mail 46 Jackson Wilson. Jack is a student in Canada. He wants to 47 Chinese.

Dear friend,

My name is Jack Wilson. I am from Toronto, 48 . I am 13 years old. I’m a schoolboy. I know China is a great 49 . I want to make friends(交朋友) 50 China and I want to learn Chinese.

There are nine students in 51 class. My clasates are from six countries. They are learning English. My parents are from France. They 52 French. I speak English and French. There are many Chinese 53 in Toronto. I want to learn Chinese , 54 I don’t he any Chinese textbooks. Could you 55 me ?

Please write back soon. Thank you.

Jack Wilson

( ) 46. A. for B. to C. from D. of

( ) 47. A. teach B. learn C. speak D. know

( ) 48. A. Canada B. Japan C. Australia D. France

( ) 49. A. city B. country C. people D. school

( ) 50. A.on B. at C. in D. to

( ) 51. A. his B. your C. my D. their

( ) 52. A. speak B. l C. say D. talk

( ) 53. A. there B. here C. peoples D. students

( )54. A. so B. and C. for D. but

( ) 55. A. give B. C. write D. lee

三. 阅读理解(3×10)

(A)

Mr. and Mrs. Smith come from London, England. They work in China now. They teach English in a middle school . They like their work very much. They he two children, Sam and Sue. They are both in Shanghai with their parents. Mr. Smith can’t speak Chinese, but his wife can speak it very well.

Mr. Smith likes swimming and reading, and Mrs. Smith likes swimming and reading, too. They often go swimming in the afternoon and read in the ning. Sam and Sue like playing s with Chinese boys and girls.

Sam’s uncle works on a farm near London. He likes swimming, too. He wants to work in Shanghai, but he can’t speak Chinese. So he is still(仍然) there and goes to Chinese classes ry day.

56. Where are Sam and Sue from ?

A. Canada B. America C. England D. Australia

57. What sport does Mrs. Smith like ?

A. Swimming B. Running C. Playing basketball D. Walking

58. What does Sam’s uncle do ?

A. A teacher B. A worker C. A farmer D. An actor

59. What does Sue like ?

A. Swimming and reading B. Playing comr s

C. Watching TV D. Playing s

60. ________ are in different countries now.

A. Mr. and Mrs. Smith B. Mrs. Smith and her children

C. Mr. Smith and his brother D. Mr. Smith and his children

(B)

A French woman goes to London to see her son. She knows a little English, but can not speak it very well.One day she goes to a shop.She wants to buy a hen(母鸡) for supper.But she can not remember(记得) the English word for “hen”.Just then the shop waiter asks her: “Can I you?”“I want the egg’s mother ”the woman answered.The shop waiter does not understand(明白) her and says:“Sorry, we he not any bigger(较大的) eggs.”Suddenly(突然) the woman remembers(记起) the word “cock(公鸡)”.She says:“I want the cock’s wife!”This time the shop waiter laughs(笑).

61.The woman goes to London_______.

A. to learn English B.to he supper C.to see her son

62.The woman________.

A.only knows a little English B.knows much English C.doesn’t know English

63.The shop waiter think the egg’s mother must be”_______”.

A.bigger eggs B.the cock’s wife C.hens

64.The shop waiter laughs because he ________.

A.understand the woman at last B.think it is interesting(有趣的)

C. does not understand the woman

65. Is the woman clr(聪明的)?

A. Yes, he is B. No, she isn’t C. Yes, she is

(C)

Dear Peter,

I know that you are coming to Luzhou next week. Welcome to my home. Let me l you the way to my house. You will live in Changjiang Ho on Changjiang Street , won’t you ? OK ! Changjiang Street is a famous and busy street in our city. You can start from your ho and go straight along Tongcheng Street. You pass two all streets: Hongxing Street and Lujiang Street. When you see an overbridge (高架桥),please turn right and go along Huangcheng Street. It’s an old street, but it’s clean. There’s a big park on the street. It’s beautiful and clean. When you get to Jinzhai Street, please turn left and go along it. You will pass Wuhu Street and Tunxi Street. Then you can see a KFC on your right. There’s a supermarket across from the KFC and my house is near the supermarket. There’s a pay phone in front of the KFC. You can call me there and I will go to meet you.

I hope you will he a good trip.

Yours,

Li Li

66. Where will Peter live in Hefei ?

A. Li Li’s room B. Changjiang Ho C. Anhui Ho D. His friend’s house

67. How is Changjiang Street by Li Li’s words ?

A. It’s beautiful . B. It’s famous. C. It’s old. D. It’s famous and busy.

68. How many streets are mentioned (提及) in this letter ?

A. 6 B. 7 C. 8 D. 9

69. How is the park ?

A. It’s old but it’s clean. B. It’s on Tongcheng Street.

C. It’s beautiful and clean. D. It’s quiet and dirty.

70. Where’s the KFC ?

A. It’s near Li Li’s home. B. It’s between the supermarket and the pay phone.

C. It’s across from the supermarket. D. It’s across from the pay phone.

七年级英语下册次月卷 2

班级 姓名 得分

Ⅰ.单项选择。(15分)

( ) 1. - Where is your pen pal from?

- _________

A. She's from Cuba.B. It's from Canada.

C. They're from Canada.D. We're from the U.S.A.

( ) 2. - Is this your book, Jane?

- Yes, it is _________ book.

A. hisB. herC. hersD. my

( ) 3. - May I speak to Jim, please?

- Sorry, _________ now.

A. I'm not JimB. this is meC. this isn't meD. he isn't here

( ) 4. - _________ does your friend Jianjian look like?

- He is tall and has black hair.

A. HowB. WhatC. WhichD. Why

( ) 5. - What does your father do?

- _________

A. He's old.B. He's tall.

C. He likes me.D. He's a cook.

( ) 6. - _________ is this pair of pants?

- It's 80 yuan.

A. How manyB. How longC. How muchD. How often

( ) 7. - What about going for a picnic with us?

- I'm sorry I can't. My mother isn't _________. I he to cook.

A. outB. inC. atD. for

( ) 8. - _________ do you like this blue sweater?

- Not bad.

A. WhatB. How aboutC. HowD. What about

( ) 9. - Would you like to he lunch with me?

- _________

A. Yes, I'd love to.B. No, you can go.

C. Yes, I he soming to do.D. You're wrong.

( ) 10. - These clothes are very n. Could I _________?

- Sure.

A. try it onB. try them onC. try on itD. try on them

( ) 11. - Don't forget _________ your school bag home.

- All right, see you then.

A. takeB. to takeC. bringD. to bring

( ) 12. - Do you he any time this afternoon?

- Yes. _________

A. What are you?B. What's going?

C. What's up?D. What's the matter?

( ) 13. - _________

- His forite color is white.

A. What's your forite pet?B. What's his forite pet?

C. What do you like?D. What's your uncle's forite color?

( ) 14. - What can I do for you, madam?

- Well, I'm __________ looking. Thanks.

A. justB. goodC. soD. only

( ) 15. - Can I you?

- _________

A. That's good.B. OK, I'll take it.

C. Yes, I'd like a coat.D. No, you can't.

Ⅱ.情景交际。(5分)

从方框中选择适当的句子补全对话。其中有两项是多余的。

Lucy: Hello!

Jack: Hello! Is that Lily?

Lucy: 16 Who is this?

Jack: This is Jack, Lily's clasate.

Lucy: This is Lucy, Lily's sister. Can I take a message?

Jack: Thanks. 17

Lucy: Sure. Does she know your phone number?

Jack: 18

Lucy: What's your phone number?

Jac---OK. By the way, are there any specials in your restaurant today?k: 19

Lucy: (1321)2588-7678. OK, 20

Jack: Thank you.

Lucy: That's OK. Bye.

Jack: Bye-bye.

A. Do you know her?

B. I'll l her about it.

C. No, she doesn't.

D. I'm Jack.

E. My phone number is (1321)2588-7678.

F. Sorry, she isn't in now.

G. Could you ask her to call me back tomorrow morning?

16. _________17. _________18. _________19. _________20. _________

Ⅲ.完形填空。(15分)

On Sunday morning, it's fine. My friends Li Dong, Wang Lei, Robert and I would like to

21 to the West Hill. We plan to he a 22 . Li Dong wants to buy a bag of salt and

23 vegetables. Wang Lei wants to buy four 24 milk and some hamburgers. I he no many cups. I want to buy some 25 in the shop. What about Robert? He's new here. He 26 from Canada. He has a big nose and two big blue eyes. His hair is light brown. He likes to go 27 on Saturday(星期六) with his mother. Now he isn't at home. So I call Robert with my cell phone, " 28 forget to take the guitar." Of course, he doesn't forget 29 his white dog. He likes the pet. We like 30 kites on the hill. I think it's fun.

( ) 21.A. goB. goingC. to go

( ) 22.A. partyB. picnicC. meeting

( ) 23.A. anyB. someC. a little

( ) 24.A. bottleB. bottlesC. bottles of

( ) 25.A. knivesB. cakesC. cups

( ) 26.A. comeB. comesC. are

( ) 27.A. shopB. shoppingC. swim

( ) 28.A. Doesn't B. Don'tC. Isn't

( ) 29.A. takesB. takeC. to take

( ) 30.A. fliesB. flyingC. playing

Ⅳ.阅读理解。(30分)

AKangkang is a Chinese boy. He is a student. He is 14. Kangkang likes to make friends(交朋友). Now he receives(收到) a letter from Sam, his pen pal in England. The letter is in English. Sam is a student, too. He speaks English. He likes China. He wants to visit the Great Wall.

( ) 31. Kangkang is a _________.

A. teacherB. studentC. doctor

( ) 32. The letter is in _________.

A. ChineseB. JapaneseC. English

( ) 33. Sam lives in _________.

A. ChinaB. the U.S.A.C. England

( ) 34. _________ wants to visit the Great Wall.

A. SamB. KangkangC. Kangkang's brother

( )35. Sam is Kangkang's _________.

A. clasateB. pen palC. teacher

BMrs. White doesn't like fish at all, but Mr. White likes fish very much. He buys some fish and takes it home. Mrs. White sees the fish and she thinks, "Good! I'll ask my friends to he lunch and we can he the fish. They like fish very much."

Mr. White comes home in the ning. He can't find his fish. Mrs. White says, "Oh, your cat eats it."And she gives Mr. White some bread for supper. Mr. White is not happy at all. He takes the cat and Mrs. White goes to a shop. He says to her, "Now you see, my cat is one kilo and the fish is one kilo, too. The cat is here. But where is my fish?"

( ) 36. What does Mr. White like very much?

A. Bread.B. Chicken.C. Fish.D. His cat.

( ) 37. Does Mrs. White like fish very much?

A. Yes, she does.B. No, she doesn't.

C. Yes, she likes.D. No, she not likes.

( ) 38. Who does Mrs. White ask to he the fish?

A. Mr. White.B. The cat.C. Her friends.D. Her mother.

( ) 39. Does the cat eat the fish?

A. Yes, it does.B. Yes, he does.

C. No, the cat doesn't.D. No, it doesn't.

( ) 40. What does Mr. White he for supper?

A. Some bread.B. Some fish.C. Some jiaozi.D. Some eggs.

COn Sunday, Kangkang goes out for a picnic with his friends. Maria's pet dog goes, too. They take some food there. Jane likes singing very much. She wants Kangkang to sing a song with her, but he has no time. Kangkang has to cook. Wang Ping and Michael would like to him. St carries water for them. And Maria flies a kite with her pet dog. They are all very happy!

( ) 41. Who does Kangkang goes out for a picnic with?

A. His clasates.B. His friends.C. His sister.

( ) 42. What does Jane want Kangkang to do?

A. Cook food.B. Fly a kite.C. Sing a song.

( ) 43. Wang Ping and Michael would like to _________.

A. cookB. carry waterC. sing a song

( ) 44. _________ fly a kite.

A. MariaB. The pet dogC. A and B

( ) 45. They _________ on Sunday.

A. go fishingB. go for a picnicC. go shopping

Ⅴ.词汇。(10分)

(A)根据汉语提示,完成下列句子。

46. - _________ _________(多少钱) are the apples?

- They are 20 yuan.

47. - _________ you _________(有空的) tomorrow morning?

- Yes, what's up?

48. I _________ _________(害怕) I can't get home in time.

49. - Hello!

- Hello! May I _________ _________(和……说话) Tom?

50. I _________ _________(不得不) look after my grandmother at home.

(B)用所给单词的`正确形式填空。

51. Don't forget _________(bring) your kite when you go for a picnic.

52. - Why not _________(take) it? It looks very n.

- All right.

53. - Would you like _________(go) to the zoo?

- Yes, I want to see animals.

54. - What about (fly) kites this afternoon, Kangkang?

- Good idea.

55. I'll l _________(he) about it.

Ⅵ.综合填空。(10分)

看下面家谱,补全短文,每空一词。

Look at my 56 tree, please. I am Mike. I'm from the U.S.A. I he a big happy family. My wife(妻子) 57 is a nurse. We he 58 kids. 59 is our son and Joy is our 60 . They study at No.1 High School. My father's name is Bill and my 61 name is Hellen. My 62 are old. John is my 63 . He is a doctor. Kate is my wife's 64 . She is a worker. My wife's 65 name is Emma. My wife's father's name is Peter. They're old, too. They stay at home.

56. ___________57. ___________58. ___________59. ___________60. ___________

61. ___________62. ___________63. ___________64. ___________65. ___________

Ⅶ.书面表达。(15分)

根据下面所提供的信息,写一篇50-60词的短文,介绍你的笔友。

姓名:Billy 年龄:12岁 国籍:美国 父亲:医生 母亲:教师

爱好:养宠物狗 最喜欢的食物:汉堡包

------------------------------------------------------------------------------------------------------------------------------------------------------------------------------------------------------------------------------------------------------------------------------------------------------------------------------------------------------------------------------------------------------------------------------------------------------------------------------------

七年级英语试题答题卷

班级 姓名 得分Ⅰ.单项选择。(15分)

12345678910

1112131415

Ⅱ.情景交际。(5分)

1617181920

Ⅲ.完形填空。(15分)

21222324252627282930

Ⅳ.阅读理解。(30分)

31323334353637383940

4142434445

Ⅴ.词汇。(10分)

46474849505152535455

Ⅵ.综合填空。(10分)

56575859606162636465

七年级英语试题

参:

Ⅰ. 1-5A D D B D

6-10C B C A B

11-15B C D A C

Ⅱ. 16-20F G C E B

Ⅲ. 21-25A B B C C

26-30B B B C B

Ⅳ. 31-35B C C A B

36-40C B C D A

41-45B C A C B

Ⅴ. (A)46. How much47. Are; free A D C C B B B B A A B C C B D A C B48. am afraid

(B) 51. to bring52. take53. to go 54. flying55. him

Ⅵ. 56. family57. Jean58. two

59. Tom60. Daughter 61. mother's

62. parents63. brother64. sister

65. mother's

七年级下册英语单元练习题

61.__________ 62. __________ 63. ___________ 64. ___________ 65. __________

七年级(下)英语期末测试卷

6. He is Jack Smith. (变为同义句)

I. 单项填空

( ) 1. ---Hi, John. Where did you ______ your winter holiday this year?

---Well, we stayed in Harbin.

A. he B. spend C. use D. cost

( ) 2. Did you see Mary’s sister? What does she ______?

A. be like B. is like C. look like D. like

( ) 3. The book describes a girl ______ Al.

A. calling B. name C. called D. to call

( ) 4. ---Excuse me, sir. Would you like to he a look at the ______ now?

A. order B. dish C. cook D. menu

( ) 5. He decided to do it by himself and wouldn’ listen to my ______.

A. exercise B. suggestion C. word D. music

( ) 6. When the ephone rang, he stopped ______ the novel and got up to answer it.

A. to read B. reading C. reads D. be read

( ) 7. Look! Two students are standing ______ the class a dialogue.

A. in the front of B. in front of C. next to D. outside

( ) 8. He is a good basketball player ______ the city’s basketball team.

A. for B. on C. in D. with

( )9. This is a difficult problem. Let’s he a meeting to ______ it.

A. report B. watch C. show D. discuss

( ) 10. ---Where are my toys, mum? ---Oh, I ______ them under the bed.

A. left B. put C. took D. made

( ) 11. If you are ______, we can stop to he dinner.

A. tired B. angry C. sleepy D. gry

( ) 12. ---Can you l me how I can ______ my spoken English?

---Pract it ry day.

A. learn B. improve C. study D. speak

( ) 13. ---I want to know your idea about ______.

---Well, I think that’s soming popular.

A. museum B. fashion C. soap opera D. culture

( ) 14. At the class meeting, many students ge their ______ about family rules.

A. sounds B. opinions C. situations D. minds

( ) 15. ---What does your father ______ your new jeans?

---He doesn’t like it at all.

A. like of B. look at C. look for D. think of

II. 补全对话

(A) 从方框内所给句子中选择恰当的句子完成下面对话。

A: Good afternoon! Can I you?

B: Yes, please! ___1___

A: We he many popular kinds, like Nike, Adidas and so on.

B: I like the things made in China. ___2___

A: Yes, Li Ning is also popular here. What about this pair? ___3___

B: Well, I don’t want a new kind. But I’d like a practical(实用的) kind.

A: This pair looks n. It came to the market half a year ago. It sells well.

B: ___4___

A: 350 yuan.

B: ___5___ But I’ll take them.

A. That’s a bit expensive. B. It’s a new kind in the market.

C. I’m looking for a pair of sports shoes. D. Do you he Li Ning sports shoes?

E. Sorry, we don’t he any. F. How much are they?

G. I want a pair of Nike shoes.

(B)在下面对话的空白处填入适当的话语(可以是句子、短语或词),使对话完整。

A: Hello, can I you?

B: Hello. I want to order a pizza.

A: Sure.What ___1___ would you like?

B: I’d like a large pizza

A: ____2____ on it?

B: Tomatoes and mushrooms.

A : OK. Anything else?

B: No, thanks.

A: Your ____3____, please?

B: 65 Fifth Avenue.

A: And your ____4_____?

B: It’s 6611019 .

A:That’ll be 2 dollars.And we’ll deliver your pizza in 20 minutes.

B: Thanks a lot.

A: ____5_____.

III. 完形填空

A man went into a restaurant one day. He sat down at the __1__ near the window. A waiter came up to __2__ and said, “Can I you, sir?” The man said, “Can I see the __3__, please? I would like a __4__ meal(餐).” Then the man __5__ a lot of n dishes. The waiter got them for him at once. The man was eating his meal. At that time a boy came in and sat down next to the man. He asked the __6__ to give him an -cream. The man says, “I go out __7__ a news.” Then he went out. After the boy ate his -cream up, he __8__ and went to the door.

“Father? No, I don’t __10__ him. I meet him in the street. He says he can give me an -cream if I come here on time.”

( ) 1. A. desk B. chair C. sofa D. table

( ) 2. A. her B. him C. them D. it

( ) 3. A. news B. food C. menu D. pr

( ) 4. A. cheap B. good C. lot D. bad

( ) 5. A. ordered B. made C. cooked D. ate

( ) 6. A. waiter B. man C. boy D. father

( ) 7. A. with B. as C. for D. and

( ) 8. A. sat down B. stood up C. went to bed D. had a rest

( ) 9. A. Sorry B. Excuse me C. Hello D. OK

( ) 10. A. like B. C. know D. l

IV. 阅读理解

Passage 1

Mr Green kept dreds of chickens on his farm. He sold the eggs and the meat to the local people. He got quite a lot of money from it, but he lived in a very hot part of the country, and he found that his hens laid(下) few eggs in summer. So he decided to put air conditioners(空调) into the chicken house in order to let the hens lay well all the year. By this he could he more eggs and get more money.

After Mr Green called the air conditioner company, a worker from the company came to see Mr Green. He thought that he might make Mr Green buy more air conditioners.

“Your wife may be very happy if you buy air conditioners for your house,” he said to Mr Green. But Mr Green was not interested. “My wife doesn’t lay eggs,” he said.

( ) 1. Mr Green ________.

A. was a fruit farmer B. sold air conditioners

C. sold vegetables D. was a chicken farmer

( ) 2. His hens laid few eggs in ________.

A. summer B. spring C. winter D. autumn

( ) 3. He thought the hens didn’t lay many eggs because they were ______

A. old B. thin C. too hot D. young

( ) 4. He wanted to put air conditioners into ______.

A. his off B. the bedroom

C. the chicken house D. the living room

( ) 5. The underlined word it means ________.

A. the farm house B. selling eggs and meat to other people

C. meat D. living in a very hot part of the country

Passage 2

( ) 6. You can he breakfast ________.

A. at 7:00 in the morning B. at 9:00 in the morning

C. at 9:45 am D. at 8:00 am

( ) 7. You can go to the coffee house at ______.

A. 10:00 B. 11:30 C. 1:00 am D. 11:00

( ) 8. If you want to he breakfast in your room, you need to ________.

A. fill in a card and put it outside B. make a ephone call to the waiter

C. find a waiter and l him

D. buy your breakfast in the dining room and bring it back to your room

( ) 9. Which one of the following is RIGHT?

A. You can see a film on Monday ning.

B. The film begins at 19:00 ry ning.

C. You can buy some things in the ho shop at 16:30.

D. You can he dinner at 9:00 in the ning.

( ) 10. Dining room is a room for ________.

A. washing clothes B. hing meals

C. waiting for rooms D. dancing

Passage 3

Welcome to Parkside, a very famous area of London. Your walking tour(游览) begins on First Street at the Windsor Ho. Turn left out of the ho, and walk down to Stanley Street. On the corner you will see the historic Parkside library. Many people think this is a very beautiful building in Parkside. Cross first Street, but be careful, because it is a very busy street. Walk down Stanley Street to Second Street. On the corner of Stanley Street and Second street you will find the Grand Ho. This famous ho is 100 years old. Turn right into Second Street. Next to the Grand Ho you will see the Parkside Museum. This is the oldest(最古老的) building in the city. Look across the street and you will see Garden Park. Next to Garden Park, on the right, is the City Zoo. Keep walking down Second Street to Walker Avenue. On the corner of Second Street and Walker Avenue is the Tourist Shop. Here you can buy tcards of Parkside. Continue walking down Walker Avenue, and you will find yourself back at the Windsor Ho.

( ) 1.This passage is _______.

A. a news article B. a tourist guide

C. an aertisement D. a story

( ) 2. Parkside is _______.

A. a city B.a town C. part of a city D. a country

( ) 3. The oldest building in Parkside is a ________.

A. museum B. library C. ho D. zoo

( ) 4. Parkside is a good place for people to enjoy ________.

A. shopping B. sightseeing C. sports D. food

( ) 5. Parkside is in _______.

A. China B. Britain C. Japan D. America

Passage 4

In the world today, all of the people need to relax. We cannot work all the time if we want to be healthy and enjoy life.

Everyone has his own way of relaxing. A very popular way is to take part in sports. There are team sports, such as basketball and soccer. There are also individual sports, such as swimming and running. Skating and mountain climbing are the most popular recreation for people who like to be outdoors.

Some people enjoy sports but don’t want to take part in them. They like watching sports s on TV or listening to them on the radio. So these people like some ways of indoor recreation, such as watching TV, singing and dancing.

Different recreations make people he healthy body and mind. They know when to work and when to relax. When they feel relaxed, they can he much energy(精力) for their work. So people should(应该) know how to work, but they should also know how to relax.

( )1. A very popular way of recreation is ______.

A. doing sports. B. watching TV

C. sleeping D. singing and dancing

( )2. People want to take part in sports to __________.

C. find a good job D. make money

( )3. Which of the following is an outdoor recreation?

A. watching TV B. singing

C. listening to the radio D. skating

( )4. What do you think the phrase “all the time” mean in Chinese?

A. 寻常地 B. 间歇地

C. 一直地 D. 努力地

( )5. The passage mainly ls us that__________.

A. basketball is a kind of team sport

B. ryone liks doing sports in their free time

C. different people he different ways of relaxing

D. indoor recreation is important

Passage 5

Dick works in an off in the city. He works very hard and really looks forward to his holiday.

He usually goes to the seaside(海边), but this year his friend ls him to go to the farm for a holiday. There’s good food and clean air. People can ride horses, he a walk and go fishing there.

“That’s a good idea.” he thinks, “I’ll spend a month on the farm. I’ll enjoy horse riding, walking and fishing. I’ll make a change. He is ready to spend his holiday on the farm. Then, he lees for the farm. Four days later, he comes back home. “What’s wrong with the farm?” his friend Jim asks, “Did you enjoy your country life?” “Yes, country life was fine,” Dick says, “but there’s another problem.” “Oh? What?” Jim asks. “Well,” he says, “the first day I was there, a sheep died, and we had roast lamb(烤羊) for dinner.” “What’s wrong with that?” Jim says, “Fresh(新鲜) meat is the best.” “ I know, but on the second day a cow died, and we had roast beef for dinner.” “That’s great.” Jim cries. “You don’t understand,” Dick says. “On the third day a pig died and we had roast pork for dinner.” “A different roast ry day,” Jim says, “why are you still complaining(抱怨)?” “Let me finish.” Dick says, “On the fourth day, the farmer died and I didn’t dare(敢) to stay for dinner!”

1. Where does Dick usually go for a holiday?

_______________________________________

2. Why does Dick want to go to the farm for a holiday?

_______________________________________

3. How many days does Dick spend on the farm?

_______________________________________

4. When did Dick eat the roast beef for dinner?

_______________________________________

5. Why does Dick came back so early?

_______________________________________

V.单词拼写

1. Yesterday I went to the supermarket and ________(预订) a new kind of bike.

2. On his way ________(往回去) home, he lost his key.

3. I ________(真正地) don’t agree with you.

4. Is there ________(任何事物) else in the bag?

5. I usually he some ________(粥) for breakfast before I lee for work in the morning.

6. The polman ________(阻止) a black car and asked the driver to show his ID card.

7. Mrs Green is medium ________(体型) and has cy blonde hair.

8. There are five ________(特色菜) in this restaurant. It’s very crowded.

9. We had great fun _______(参观) the Great Wall last Friday morning.

10. He is ________(一直) thinking of others first and then himself.

VI. 从方框中选择适当的词或短语,并用其正确形式填空

with, take, teach, because, young, middle, but, well, go for a walk, play with

Tom likes walking. He ___1___ after his father comes back from work. Tom likes water very much. H often ___2___ his toys in the water, so he is wet(湿的) all over sometimes. His mother says, “Are you from fish?” Tom is very glad, “Yes, I like fish very much ___3___ fish can swim.”

One Sunday morning, Tom went to a park ___4___ his father. There was a swimming pool in the ___5___ of the park. Tom asked his father to ___6___ him to the pool. “Why do you want to go there?” asked his father.

“Because I want to learn swimming,” answered Tom. “And you can ___7___ me.”

“You are too ___8___, and I’m sorry I can’t swim,”said the father.

“You can’t swim? Jack’s father swims very ___9___. Why can’t you swim?”

“Because his father likes eating fish,” said the father.

“Oh, I see.” said Tom. “___10___ you like eating chicken very much. Can you lay eggs(下蛋)?”

VII. 翻译句子,每空只填一个单词

1. 假如你想要身体健康,那你得多锻炼。

You ________ ________ take more exercise ________ you want to be ________.

2. 走到动物园,花了我们一个半小时

We ________ one and a half hours ________ to the zoo.

3. 他正在考虑如何才能解出这道难题。

He is ________ about ________ he can work out the hard problem.

4. 当你穿这条繁华的街道时,不要跑。

________ ________ when you ________ the ________ street.

5. 汤姆通常在外面吃饭,因为他不会做饭。

Tom usually ________ ________ because he ________ ________.

6. 我这里正下着雪呢,你那边的天气如何?

________________ here. ____________________ there?

7. 琳达说迈克在他们学校篮球队很受欢迎,他长得什么样子?

Linda says that Mike _____________ in the school basketball team. _________________?

8. 我们餐馆有许多种类的面条。你想要多大碗的面条?

Our restaurant has _________________ noodles. ______________________________?

9. 我真的忍受不了下雨天,因为那让我感觉乏味。

I really can’t __________________ because they _______________________ bored.

10. 不要在教室里面打闹。我认为你应该先学习一下学生守则。

_______________ in the classroom. I think you should ________________ first.

VIII. 书面表达

以“My Family Life with a Comr”为题目,写一篇短文。词数80个左右,内容包括你和你的家人用电脑做什么,电脑给你的家庭带来了什么影响等。

My Family Life with a Comr

_____________________________________________________________________________________________________________________________________________________________________________________________________________________________________________________________________________________________________________________________________________________________________________________________________________________________________________________________________________________________________________________________________________

七年级(下)英语期末测试卷

I. 1-5 BCCDB 6-10 BABDB 11-15 DBBBD 16-20 CADBA

II. 1-5 CDBFA

6. size 7.What do you like 8. address 9. phone number 10. You are welcome

IV. 1-5 DACCB 6-10 DCACB 11-15 BCABB 16-20 AADCC

21. He usually goes to the seaside.

22. Because he wants to enjoy horse riding, walking and fishing.

23. Four days.

24. On the second day of his holiday.

25. Because the farmer died on the fourth day and he didn’t dare to stay there.

V. 1. ordered 2. back 3. really 4. anything 5. porridge 6. stopped 7. build 8. specials 9. visiting 10. always

VI. 1. goes for a walk 2. plays with 3. because 4. with 5. middle 6. take 7. teach 8. young 9. well 10. But

VII. 1. he/need, to, if, healthy 2. spent, walking 3. thinking, how 4. Don’t, run, cross, busy

5. eats, outside, can’t, cook 6. It’s snowing, How is the weather 7. is very popular, What does he look like 8. many kinds of, What size bowl of noodles would you like 9. stand rainy days, make me feel 10. Don’t fight, learn the students’ rules

VIII. One sible version:

My Family Life with a Comr

Comr is popular in our modern life. There is a comr in my family, too. My parents bought it three years ago. They put it in the sitting room.

Father is an engineer. He starts the comr after supper ry ning and reads sports news on the Internet. Sometimes he looks for some rmation. Mother is a teacher. She likes reading, so she often reads e-books on it. I often watch movies on it or play comr s. It is great fun!

Now the comr becomes one of our best friends.

谁有七年级上的英语期末试题??急啊!!!!!!!

A. keep healthy and enjoy life B. make friends

部分 听力(20分)

Ⅰ.将你听到的字母的序号写在题前的括号内(5分)

( )1.A.D T B.B T C.P T

( )2.A.N M B.M N C.L N

( )3.A.S X B.X S C.S L

( )4.A.G J B.J G C.J K

( )5.A.Q U B.U Q C.W Q

Ⅱ.将你听到的单词或数字的序号写在题前的括号内(5分)

( )6.A.383838 B.338833 C.333888

( )7.A.class B.glass C.glasses

( )8.A.football B.volleyball C.basketball

( )9.A.100156 B.101056 C.100516

( )10.A.you B.yours C.your

Ⅲ.根据你听到的问话,选择正确的,将序号写在题前的括号内(5分)

( )11.A.Yes, it is. B.Yes, there is.

C.Yes, I can.

( )12.A.He works in a factory.

B.He has supper at sn.

C.He often watches TV at home.

( )13.A.I want a cup of te

B.I'm from Tianjin.

C.I can speak English.

( )14.A.I'm fine, thanks. And you?

B.Not at all.

C.Here it is.

( )15.A.Yes, she does.

B.No, she doesn't.

C.Yes, he does.

Ⅳ.听后完成下列文章(5分)

This is a picture 16 the bedroom. It is not 17 . The twins live here(双胞胎住在这). There is a 18 on the floor. Their shoes are under the desk. Their trousers 19 on the bed. Where're they? Now it's nine thirty. They are 20 school now.

第二部分 笔试(80分)

Ⅴ.单词辨音(2.5分)

从四个单词中,找出划线部分发音不同的:

21.A.name B.he C. D.cake

22.A.go B.so C.do D.no

23.A.pear B.wear C.bear D.near

24.A.bowl B.grow C.yellow D.down

25.A.hey B.ready C.speak D.bread

Ⅵ.单词拼写(2.5分)

从A,B,C,D四个选项中,找出正确的拼写形式:

26. zer __

A.e B.a C.u D.o

27.d___ty

A.e B.a C.u D.i

28.n __ber

A.am B.em C.un D.um

29.j __p

A.ee B.ie C.ea D.ii

30.bl __se

A.au B.ou C.uo D.ao

Ⅶ.词形转换(10分)

根据括号中的要求,写出下列单词的相应形式:

31.man(复数)____ 32.new(反义词)____

33.box(复数)____ 34.white(反义词)____

35.women(单数)____ 36.know(同音词)____

37.these(单数)____ 38.stand up(反义词)____

39.that is(缩写)____ 40.come(反义词)____

Ⅷ.选择填空(20分)

41.The man is ____father.

A.Kate B.Kate's

C.she's D.Kate's

42.How old ____?

A.are you mother B.are your mother

C.your mother is D.is your mother

43.---- Who's that girl?

---- She's ____.

A.my sister B.ten

C.in red D.n

44.I go to bed____9:15 in the ning.

A.in B.at

C.under D.on

45.Tom is ____English boy. He's in ____Class Four, Grade Three.

A.an; the B.an; ×

C.a; × D.×; ×

46.Let me ____it.

A.to do B.do

C.do to D.do the

47.---- Is this box hey or light? --____.

A.Yes, it is B.No, it isn't

C.It's light. D.Yes, it's hey

48.---- What colour is that hat?

--____.

A.That's blue. B.It's a blue

C.It's blue D.Its blue

49.Hello, Jim! N____ you!

A.see B.seeing

C.sees D.to see

50.____ turn is it now?

A.Who's B.Whose

C.Who D.Whom

51.The man ____the bike is our teacher.

A.at B.in

C.on D.by

52.I3. Andy是你的一位好邻居。他总是帮助你学英语,你的自行车坏了,他乐意帮你修理。 am ____.

A.an Chinese B.Chinese

C.the Chinese D.one Chinese

53.---- ____is your father?

---- He is all right.

A.How B.What

C.Where D.Who

54.---- Are you a student?

---- ____.

A.Yes, I am B.No, you aren't

C.Yes, you are D.Yes, I'm

55.Please look ____the picture on the wall.

A.in B.at

C.to D.on

56.---- Hello. Here is your pencil-box. ____very much.

---- ____.

A.Thank; That's right

B.Thank you; That's all

C.Thank you; You're welcome

57.---- He you any sisters?

---- ____.

A.Yes, I he B.No, hen't

58.---- ____that girl?

---- She's my sister.

A.Where's B.Which is

C.Who's D.Whose

59.---- ____are we?

---- We're in Hubei.

A.What B.Who

C.Where D.How

60.---- ____?

---- How do you do?

A.How do you do

B.How are you

D.How old are you

Ⅸ.按要求改写句子(10分)

61.There's some milk in the glass.(变否定句)

________________________

62.Sometimes he goes shopping with his mother(就划线部分提问)

_________________________

63.They he lunch in the middle of the day. (就划线部分提问)

_________________________

64.Mr White is about thirty.(就划线部分提问)

_________________________

65.He carries one friy in his hand. (变复数句子)

_________________________

Ⅹ.连词成句(5分)

66.jeep, green, is, their(.)

_________________________

67.am, in, I, not, two, class(.)

_________________________

68.his, are, these, pictures(.)

_________________________

69.colour, dress, is, Kate's, what(?)

_________________________

70.that, is, who, old, woman(?)

_________________________

Ⅺ.选择适当的词,填在相应的空处(10分)

Now look 71 the picture. The farm in the picture is in the 72 . It is not cold today. The sun is in the sky. There's a farm-house near the river. There are many tall trees 73 the farm-house. In the field there are some all trees. They are some apple trees. There are a lot of apples 74 them. There are many 75 on the farm. They are horses, cows, 76 , and pigs. The man near the gate (门口) is the 77 . A woman is feeding the chickens. This is the 78 wife. (妻子)

They he two children. One is a boy. The other is a girl. They are 79 school. They study hard.

It's a happy 80 , isn't it?

71.A.in B.at C.for D.with

72.A.country B.farm C.city D.town

73.A.in front B.in C.in front of D.at

74.A.at B.from C.on D.in

75.A.animals B.pigs C.people D.cats

76.A.a sheep B.sheeps C.sheep D.sheep's

77.A.farmer B.writer C.driver D.teacher

78.A.his B.farmer's C.farmer D.farmers

79.A.go to B.for C.at D.with

80.A.house B.family C.home D.room

Ⅻ.阅读理解(10分)

选择正确的,完成句子

Look! It's a picture of a family. There is a white house in the picture. Behind the house, there are some tall trees. In front of it, there are some flowers. There is a little child in front of the house. She's about eight. She is a schoolgirl. She doesn't go to school today. She is watering some flowers. She loves flowers. She does it ry day. A man and a woman are near the house. They are the girl's father and mother. The man is mending a bike. He is a worker. He doesn't go to his factory today, too. The woman is reading a news. She is an English teacher. She speaks English very well. She is in the same school with her daughter. To day is Sunday. They are all at home. This is a very good family.

( )81.The house is in the ____.

A.country B.city C.picture

( )82.Near the house there are some ____.

A.trees B.trees and flowers C.flowers

( )83.All the people in the picture are ____.

A.free B.working C.studying

( )84.The father goes to work ____.

A.by plane B.on foot C.by bike

( )85.It is a____ family.

A.happy B.interesting C.kind

ⅩⅢ.补全对话(10分)

86.A:How are you?

B:Fine, ____And you?

A:I'm ____, too.

87.A:Excuse me! Are you Liu Wei?

B:Sorry, I am ____.

88.A:____ that over there?

B:It's a green bike.

A:Is it yours?

B:No, ____.

89.A:are you in?

B:I'm ____Class Four.

90.A:Whose shirt is this? Is it ____, Jim?

B:No, it isn't. Mine ____green, and this one is yellow.

91.A:Excuse me, ____that boy?

B:Which one?

A:The one under the tree.

B:Oh, ____my friend, Li Lei.

学期期末测试题

Ⅰ.将你听到的字母的序号写在题前的括号内(5分)

1.B 2.A 3.C 4.A 5.C

Ⅱ.将你听到的单词或数字的序号写在题前的括号内(5分)

6.A 7.B 8.C 9.C 10.A

Ⅲ.根据你听到的问话,选择正确的,将序号写在题前的括号内(5分)

11.Is this a yellow bag? (A)

12.What does Li Ying often do at home?(C)

13.Where are you from? (B)

14.How are you?(A)

15.Does she get up at six?(A)

Ⅳ.听后完成下列文章(5分)

This is a picture 16(of) the bedroom. It is not 17(big) . The twins live here(双胞胎住在这). There is a 18(ball) on the floor.

Their shoes are under the desk. Their trousers 19(are) on the bed. Where're they? Now it's nine thirty. They are 20(at) school now.

Ⅴ.单词辨音(2.5分)

21.B 22.C 23.D 24.D 25.C

Ⅵ.单词拼写(2.5分)

26.D 27.C 28.D 29.A 30.B

Ⅶ.词形转换(10)

31.men 32.old 33.boxes 34.black

35.woman 36.no 37.this 38.sit down

39.that's 40.go

Ⅷ.选择填空(20分)

41.B 42.D 43.A 44.B 45.B

46.B 47.C 48.C 49.D 50.B

51.C 52.B 53.A 54.A 55.B

56.C 57.A 58.C 59.C 60.A

Ⅸ.按要求改写句子(10分)

61.There isn't any milk in the glass.

62.What does the sometimes do with his mother?

63.When do they he lunch?

64.How old is Mr White?

65.They carry friies in their hands.

Ⅹ.连词成句(5分)

66.Their jeep is green.

67.I am not in Class Two.

68.These are his pictures.

69.What colour is Kate's dress?

70.Who is that old woman?

Ⅺ.选择适当的词,填在相应的空处(10分)

71.B 72.A 73.C 74.C 75.A

76.C 77.A 78.B 79.C 80.B

Ⅻ.阅读理解(10分)

81.C 82.B 83.A 84.C 85.A

ⅩⅢ.补全对话(10分)

86.thanks;fine 87.not 88.What's ;it isn't

89.What class;in 90.yours ;is 91.Who's;he's

七年级下英语期末试卷

A. in Asian countries B. only in English-speaking countries

七年级下英语期末试题 卷 选择题(题,共70分)

I. 听力对话理解 (共10分,每小题1分)

A) 听小对话,每段对话你将听到前面的1—3句,从每小题的三个选项中选出能完成对话一句的答语,每段对话听两遍。

1. A. Math. B. America. C. English.

2. A. I like dogs. B. Because they're cute. C. They are eating lees.

4. A. Yes, I did. B. Yes, I was. C. It was very beautiful.

5. A. Sorry, Mr Smith. B. Oh, sorry, Mrs Smith. C. Thanks.

B) 听小对话,根据你听到的内容从每小题的三个选项中选出正确,每段对话听两遍。

6. How was Emily's summer vacation?

A. It was great. B. It was boring. C. It was OK.

7. Where was Al yesterday morning?

A. At school. B. In a shop. C. At home.

8. What does Kate look like?

A. She wears glasses. B. She has short hair. C. She is good-looking.

9. What size bowl of r porridge would Victor like?

A. A large bowl. B. A medium bowl. C. A all bowl.

10. What is Tommy's brother doing?

A. Watching TV B. Playing tennis C. Playing chess

II. 听力篇章理解。(共10分,每小题1分)

听下面AB两段对话和C篇的一段独白,从每小题的三个选项中选出一个正确。对话和独白听两遍。

(A)

11. Who did Eric go fishing with?

A. His brother. B. His father. C. His grandfather.

12. Where did Eric go fishing?

A. At the West Lake. B. At the North Lake. C. At the South Lake.

13. Did Eric get a big fish last weekend?

A. Yes, he did. B. No, he didn't. C. We don't know.

(B)

14. What would the man like?

A. Hamburgers, French fries and coffee.

B. Hamburgers, tomato soup and orange ju.

C. Hamburgers, fish and orange ju.

15. How much does the man need to pay for the dinner?

A. Three dollars. B. Four dollars. C. Five dollars.

16. When does the man pay?

A. After the dinner B. Before the dinner C. During the dinner

(C)

17. Mr Hand has __________children.

A. one B. two C. three

18. Mr Hand first took his children to__________last Sunday.

A. a swimming pool B. a mountain C. the park

19. The little girl cried because __________.

A. she couldn't find her mother B. she wanted to eat cream

C. she was gry

20. Mr Hand and his children went back home__________.

A. by bus B. by taxi C. on foot

III. 单项填空。(共10分,每小题1分)

21.There is airport in Xiaoshan. _______ airport is very large.

A. a, A B. an, The C. /, The D. the, The

22. My pen pal __________ Japan.

A. is come from B. come from C. be from D. is from

23. I am __________ they can play soccer in this heat.

A. surprise B. surprises C. surprised D. surprising

24. Lily loves to __________ jokes.

A. l B. say C. speak D. talk

25. There aren’t _________ apples. Could you please buy __________?

A. any;any B. any;some C. some;some D. some;any

26. It's bad ______ you ______ read______ bed.

A. for, to , in B. to, to, out of C. to, /, on D. for, not , in

27. ----Do you know the boy ____________ a funny hat and sunglasses?

----Of course, he is a pop singer, I really like him.

A. is B. has C. wears D. with

28. It's four o'clock! Everyone ___________ a good time on the playground.

A. has B. he C. is hing D. are hing

29. ---- __________ do you____________ the colorful clothes?

----I like them.

A. How, think B. What, think of C. What, think D. How, think of

30. ----What time do you usually get up?

---- I usually _______at six, but this morning I ________ at six ten.

A. get up, get up B. am getting up, got up

C. get up, got up D. am, was

IV. 完形填空。(共10分,每小题1分)

The weather is different(不同的)in different parts of the world. In some places it is dry(干燥的), and in 31 it is humid. If the weather is too dry, the land will not be 32 for animals or plants. In humid weather there may be too much 33 . The rivers may go over their sides. The 34 may take the bridge away. If the rivers go over their sides, a lot of people may not he enough(足够的) food. If there is very dry weather for a long 35 , the river beds may be dry. In some other parts of the world the weather may be very cold. There may be much 36 . When it snows, the trees, the buildings and rything look 37 . In winter the days are short and the nights are long. 38 cold winter nights, when there are no clouds

39 winds, the sky is very clear. And the moon and stars are very beautiful. People may put on their coats and go out for long walks. When they come back to their houses, they may be happy 40 hot coffee and cakes by the fire.

31. A. other B.the other C. others D. the others

32. A. worse B. bad C. good D. better

33. A. snow B. rain C. cloud D. wind

34. A. rain B. water C. river D. wind

36. A. cloud B. rain C. snow D. wind

37. A. green B. blue C. white D. yellow

38. A. At B. On C. In D. After

39. A. and B.and or C. with D.or

40. A. to he B. to do C. to drink D. to eat

V. 阅读理解。(共30分,每小题2分)

(A)

Name Job Time Place Feeling

St Polman July 15th Mount Tai Pretty good

Henry Actor July 25th The Central Park Not bad

Nancy Student August 1st The Summer Palace Fantastic

Julie Doctor August 1st City Museum Terrible

41. What does Nancy do?

A. A doctor. B. A polwoman. C. An actor. D. A student.

42. When did Henry go to the Central Park?

A. On July 15th. B. On July 25th. C. On August 1st. D. On August 10th

43. Where did St go? w w w .x k b 1.c o m

A. To the Summer Palace. B. To the Central Park.

C. To Mount Tai. D. To City Museum.

44. How was Julie's vacation?

A. Fantastic. B. Not bad. C. Pretty good. D. Terrible.

45. Which of the following is not true?

A. Julie is a doctor. B. Henry is an actor and his vacation wasn't bad.

C. Nancy went to the City Museum on vacation.

D. Nancy and Julie went on the vacation on the same day.

(B)

Frank was clr, but he nr liked to work hard. He often said to his friends, "If you work hard, you will make a lot of money, but it is not so good. I want easy work and a lot of money. That's the most interesting thing in the world."So he could only be a thief. But he still thought it was too much work, so he o nly told his friends what to do and got money from them.

One day, Frank sent one of his friends to a very large and beautiful house. He told him to get money from that rich family. It was ning, and a man and a girl were in the room. They were playing a duet(二重奏)on a piano. Then the thief came into the house.

When he returned, Frank asked him what he had got. But he said, "I didn't take anything. That family can't he much money. You know, two people were playing on the same piano there. They did not he money to buy another piano."

46. Which is the best title for the passage?

A. Frank and His Work . B. One Piano Only.

C. The Man and the Girl. D. Easy Work and More Money.

47. From the story we know that Frank________________ .

A. had many clr friends B. was more clr than most of the people

C. didn't want to do any work D. knew many interesting things in the world

48. Frank's friends __________________ .

A. were all this B. were clr enough to do easiest work

C. usually worked as hard as Frank D. knew who were rich and who were poor

49. The thief didn't go into the beautiful house because _____________ .

A.that family was not rich B.he knew nothing about music

C.the piano was too big for him D.the man and the girl had already seen him

50. The story ls us if we want to become rich, we must ________________.

A. find an easy job B. work hard

C. he a lot of friends D. learn soming about money

(C)

I am sitting on a beach on this hot summer day and watching two children,a boy and a girl,playing in the sand.They are working hard at building a sandcastle(沙堡).Just when they are successful, a big we(波浪)comes along and puts it down.

I think the children may cry because all their hard work turns to nothing, but they surprise me. They run up away from the water, hand in hand, and sit down to build another one.

They really teach me a lesson. We can't always be successful.Being sad is doing no good.What we need to do is to start again, hand in hand with our friends.

51. What's the weather like?

A. It's sunny but cold. B. It's windy and cool.

C. It's sunny and hot. D. It's windy but warm.

52. Who is playing in the sand?

A. Two children. B . Four children.

C. Two children and the writer. D. It's not mentioned(提到).

53. What do the children do after a big we puts down the sandcastle?

A. They cry sadly. B. They just sit on the beach.

C. They start to build another one. D. They start to sing.

54. Which of the following is TRUE according to the passage?

A. The writer is a teacher. B. The we destroys(毁坏)the sandcastle.

C. The write knows the children .

D. The children are sad because they're not successful

55. What's the main idea of the passage?

A. Friends are always there with you. B. Children know more about life.

C. All of us want to be successful. D. Starting again is the best way.

松滋市2011—2012学年度下学期

七年级期末英语试题

题号 I

II

III

IV

VVI

VII

VIII

IX

总分

分数

卷答题卡

题号 1 2 3 4 5 6 7 8 9 10 11 12 13 14 15

选项

题号 16 17 18 19 20 21 22 23 24 25 26 27 28 29 30

选项

题号 31 32 33 34 35 36 37 38 39 40

选项

题号 41 42 43 44 45 46 47 48 49 50 51 52 53 54 55

选项

第二卷 非选择题(四大题,共40分)

得分 评卷人

VI. 完成句子。(共10分,每小题2分)

根据括号内的汉语和句末括号内的英语单词提示完成句子。

例如:---- What are you doing here?

----We ___________________(在等待)the bus. (wait)

: are waiting for

56. ----Oh, it's 6:30 now. ______________________________(该起床了),Jim!

----OK, mom! (time)

57. ----What are the rules at your school?

- ----Don't run in the hallways and don't_________________(上课迟到). (arrive)

58. We had fun ______________________________(打沙滩排球)last Saturday

afternoon. (play)

59. ----What did your parents do yesterday ning?

----They ________________________________(在看脱口秀)on TV in the

sitting room. (watch)

60. I often listen to English songs______________________________________

(为了提高我的英文水平) (improve)

得分 评卷人

VII. 短文填空。(共10分,每小题1分)

阅读短文,并选择方框中词的适当形式填空。

l, minute, walk, ask, on, with , this, one, stop, good

Mr Zhang is in New York 61 days. He goes there 62 vacatiion. When he goes out, he always takes a map 63 him because he doesn't know the way very 64 . But he can't find ry place on the map because some places are too all. Then he has to ask the polman for . Yesterday Mr Zhang's watch

65 , and he wanted to buy a new 66 . But he couldn't find a watch shop. He went to a polman and 67 the way. The polman said," 68 straight and then turn right. It's next to a fruit shop." Mr Zhang said, "Thank you for 69 me, sir." Ten 70 later he went to the wat ch shop.

66. _________ 67. ___________ 68. __________ 69. ___________ 70. __________

得分 评卷人

VIII. 功能阅读。(共10分,每小题2分)

阅读短文并按每小题要求答题。

Everyone has dreams(梦想). Different people he different dreams. What dreams do you he? How can you make them come true?

Some kids ① /faInd [n 5IntrIstIN weI/. They write letters to Barack Obama四、1. between and 2. like walking 3. do go 4. Would some 5. Did he. Do you know who he is? He is the 44th president()of America. He is a black man. ②He is the first black president in American history. He is also a great man. He loves ing people. People love him and they are interested in him. ③They think he can bring a new America. So they l their dreams to him. More than 4,500 kids from 5 to 12 years old all over the world write letters to Obama. Some kids can't write the words. They draw some n pictures about their dreams. Their dreams are different and interesting. Some of the dreams are very great. The dreams are "Please stop the war.","Please make the Earth greener.","Please find a job for my father.",and so on. ④ A boy n wants to he a "candy(糖果)rain".

The kids are young, but they he their own ideas about the world. It's very important for the world, n for ourselves.

71. 根据①处的音标写出英语单词:(2分,每个词0.5分)

_____________________________________________

72. 1). 在②处的划线句子中找出两个可以连读的词:______________________

2) 在②处的划线句子中哪个词的一个音可以失去爆破?____________

73. 将③处的划线部分译成汉语。

___________________________________________________

74. 根据④处的划线部分在下面的句子的每个空白处填入一个适当的词。(首字母已经给出)

The boy likes e candy very much, and he wants to he l of candy.

75. 回答问题: Everyone has dreams. Please l us your dream.

_____________________________________________________________

IX. 书面表达。(共10分)

根据表格提示,写一篇60--80词的日记。注意格式,可以适当发挥。

Date Monday, October 1st

Weather sunny, windy

Places to go the Great Wall, the Summer Palace, the Palace Museum, a Beijing Hutong

Dinner r, fish, vegetables, fruit, delicious

Feeling tired, happy

--------------------------------------------------------------------------------------------

七年级英语期末复习知识点

【典句必背】

1. —What did you do last weekend? 上个周末你做什么了?

—I did my homework. / We went boating. 我做了我的家庭作业。/我们去划船了。

2. —Who visited her grandma? 谁看望了她的奶奶?— Becky did. 贝姬看望了。

3. My sister finished high school two weeks ago. 我的姐姐两周前中学毕业了。

4. But I was so tired that I went to sleep early. 但是我是如此疲倦,以至于我很早就睡着了。

【经文】

七年级上英语期中卷

D.Thank you; No, no

英语册(上)

期中模拟测试

I. 选出划线部分读音与其他三个不同的选项: (5%)

( ) 1. A. picture B. nine C. fine D. Chinese

( ) 2. A. school B. too C. book D. two

( ) 3. A. comr B. student C. excuse D. but

( ) 4. A. eraser B. apple C. grade D. cake

( ) 5. A. thinks B. thank C. these D. thir

( ) 6. A. boats B. cats C. students D. mouths

( ) 7. A. pen B. American C. eraser D. twelve

( ) 8. A. those B. clock C. box D. on

( ) 9. A. middle B. right C. miss D. sister

( ) 10A. ruler B. cup C. mother D. bus

II. 单词拼写: (10%)

1. 根据规律默写数字:

Two _________ _________ ________ ten ________ __________

_________ Three ________ ________ _______ eln ________

2. 根据句意及给出的首字母填写单词:

1) Is this your car l__________ number?

2) We study in the same m__________ school.

3) These two boys are t___________. They look the same.

4) Who is on d_________ today?

5) I can see two beautiful birds in the p___________.

6) Listen to this E________ song. It’s n.

7) W_________ to Beijing!

8) Can you s_________ your name, please?

9) –How old are you?

--It’s a s__________.

10) –What’s five m_______ two?

--It’s three.

III. 按要求变化词形: (5%)

1. class _________ (复数) 2. Japanese ______(复数)

3. here _________(反义词) 4. right _________(反义词)

5. it’s __________(同音词) 6. two __________(同音词)

7. they’re _______(原形) 8. is not ________(缩写)

9. her __________(主格) 10.they ________(形容词性物主代词)

IV. 选择最合适的:(20%)

( ) 1. I ________ a student. You _________ my teacher.

A. is, is B. are, am C. am, are D. am, is

( ) 2. This is a ____________. He is in No. 5 _____________.

A. Middle School, Middle School

B. middle school, middle school

C. Middle School, middle school

D. middle school, Middle School

( ) 3. –Is it __________ Chinese film?

--No, it is __________ American film.

A. a, an B. an, a C. a, a D. an, an

( ) 4. --Are these Japanese toys?

--_______________________

A. No, they are. They are made in China.

B. These are not Japanese toys. They are made in China.

C. No, they aren’t.

D. These are Chinese toys.

( ) 5. –Who are these ___________?

--They __________ our teacher.

A. woman, are B. women, are C. woman’s, are D. women, is

( )6. –Excuse me, are these __________English books?

--No, they are ____________ English books.

A. your, her B. my, she C. your, he D. he, her

( )7. Is this girl English _________ American?

A. and B. but C. or D. of

( )8. –What are those?

--They are _______________.

A. pencils-box B. pencil-box C. pencil-boxes D. pencil-boxes

( )9. Xiao is __________Chinese. He is ___________ Chinese boy.

A. a, a B. /, a C. /, / D. an, an

( )10. These two pens look ____________.

A. same B. his C. the same D. her

( )11. –Is Tom ________ school today?

--No, I think he is _________ home.

A. at, at B. at, in C. in, at D. in, in

( )12. What’s ten plus two? Can you ________ this question?

A. ask B. think C. see D. answer

( )13. Please look ___________ my baby tomorrow.

A. at B. for C. after D. on

( )14. –What’s that?

--__________________

A. It’s too a cat.

B. Too it’s a cat

C. It’s a cat, too.

D. It’s a cat too.

( )15. I think you are ___________. Five and six isn’t ten.

A. wrong B. right C. fine D. n

( )16. __________ is he late today?

A. What B. Why C. How D. Who

( )17. Who is the girl ____________?

A. over here B. at here C. at there D. over there

( )18. –Where is my English book?

--Sorry, I don’t ___________.

A. see B. forget C. know D. look

( )19. __________ ryone here today?

A. Are B. Is C. Am D. Does

( )20. What’s that _________ Chinese?

A. at B. for C. on D. in

V. 填入合适的介词、副词:(5%)

1. Sit __________, please.

2. Good morning __________ you!

3. What row are you ___________?

4. Hurry _________! You are late.

5. What can you see _________ this picture?

6. Is James __________ duty now?

7. Please look __________ the blackboard.

8. Tom, please look ___________ his dog.

9. Let’s listen ___________ the English song.

10. Are you __________ the same class?

VI. 根据回答,填入合适的疑问词:(5%)

1. _________ one and five? It’s six.

2. _________ is our teacher? She is in the library.

3. _________ are you today? I am fine.

4. _________ is this in English? This is a map.

5. _________ old is your little brother? He is five years old now.

6. _________ is the girl in blue? She is my good friend.

7. _________ grade are you in? I am in Grade Six.

8. _________ is her fax number? 02664791.

9. _________ is not here today? Tom.

10. _________ is his name? He is Wang Gang.

VII. 根据要求改写句子:(10%)

1. This is a pretty girl. (变为复数)

__________________________________

2. These are Japanese cars. (变为一般疑问句)

__________________________________

3. She is our new teacher. (变为否定句)

__________________________________

4. Tom is in the library. (根据划线部分提问)

___________________________________

5. Those students are Chinese. (变为单数)

___________________________________

_____________________________________

7. Is this girl your sister? (用your friend变为选择疑问句)

_____________________________________

8. Excuse, are you Mr. Jackson? (否定回答)

______________________________________

9. Is Sandy in? (同义句)

______________________________________

10. What’s one and three? (同义句)

______________________________________

VIII. 根据中文完成下列句子:(10%)

1. 这两个男孩是双胞胎,他们看上去长得一样。

These two _________ are _________. They _________ the _________.

2. 请问,这些用日文怎么说?

________ me, what _______ these ________ _________?

3. 这幅画里你能看见什么?

What ________ you _______ _________ the ______________?

4. 仔细听这首英文歌。

_________ ________ this _________ song __________.

5. 我想我不知道。

I _________ I ________ ________ the __________.

IX. 为下列表达法配对:(10%)

( )1. How is your father? A. I think she is his sister.

( )2. See you later! B. No, it isn’t.

( )3. Who is this girl? C. See you!

( )4. Glad to meet you! D. It’s Tom!

( )5. What row are you in? E. It’s over there.

( )6. Here you are. F. Well, it’s a secret.

( )7. Where is the washing room? G. He is fine. Thank you.

( )8. How old are you? H. Glad to meet you, too!

( )9. Who is on duty today? I. I am in Row Five.

( )10. Is this your ruler? J. Thank you.

( )11. Thank you very much. K. It is Ok.

( )12. I am sorry. L. It’s my pleasure.

X. 完形填空:(10%)

He _________ a middle school __________. ________ name is Zhang Gang. He is __________ Class Two, __________ Three. His English teacher ___________ Miss Liu. ____________ is a good teacher. _________ Chinese teacher is Mr. Li. _________ is a good teacher, ______________.

( )1. A. am B. is C.It is difficult for an elephant to bend(弯曲) down to reach food on the ground because of its short, fat legs. That is why an elephant has a trunk. It uses its trunk to reach food it needs. Without its trunk, an elephant would be less. are D. isn’t

( )2. A. student B. boy C. pupil D. twin

( )3. A. Her B. Their C. He D. His

( )4. A. on B. in C. for D. under

( )5. A. Row B. Class C. Number D. Grade

( )6. A. am B. is C. are D. isn’t

( )7. A. She B. He C. Her D. His

( )8. A. Her B. She C. His D. He

( )9. A. She B. He C. Her D. His

( )10A. but B. or C. and D. too

XI. 阅读理解,判断正误:(10%)

I am an American boy. My name is Bob Twist. I am in No. 2 Middle School. I am in Class Three, Grade Two. I he a Chinese friend. His name is Gao Ming. We are in the same class. He is on duty today.

( )1. The American boy’s name is Bob.

( )2. He has an English friend.

( )3. His friend is in No. 2 Middle School, too.

( )4. Gao Ming is in Class Three, Grade Three.

( )5. The American boy is on duty today.

笔试部分(85分)

二.根据句意写单词(10分)

1. Your father’s father is your g______.

2. Jenny is my pen f______.

3. I don’t he a tennis r______.

4. let’s play v_____.

5. He doesn’t play sports, he only w_____ them on TV.

6. —Let’s play soccer.

—That sounds I_______.

7.I don’t like math. It’s d______.

8.My father and mother are my p_______.

9.We should eat lots of v_______.

10.She eats lots of h_______ food ry day.

三.选择题。(15分)

( )1.______!Is he your teacher

A. Sorry B Excuse me C Ok D. See

( )2.Mr Green _____ two children.

A. doesn’t he B .he

C. doesn’t has D. don’t he

( )3.My sister Sue ____ a new coat ,but I _____

A has, don’t B. he, doesn’t

C doesn’t he ,don’t D. don’t he, does

( )4.Let’s ______

A .to play ping-pong B. playing ping-pong

C .play the ping-pong D. play ping-pong

( )5.How old is ____mother?

A. Kay B .Kay’s C. she D. he

6your dresser is n.

—_______.

A. Ok B. Yes ,it is.

C. Thank you. D. No, it isn’t.

( )7.I he ___uncle.

A. an B .a C. the D. \

( )8.— Do they he a comr?

--_______

A. No, they hen’t

B. No, they aren’t

C. No, they do

D. No, they don’t

( )9.Tom and Peter ____fruit very much.

A。likes B .eat C .like D. eats

( )10.There ___some books ,a pencil case and a

pen in his desk.

A .is B .he C .are D. don’t he

( )11._____ your bag in the desk?

A. Do B. Is C .Are D. Does

( )12.—Are ___you teachers?

- -Yes \, they are.

A. this B .that C .the D .these

( )13.—Is that your family photo?

--_____.

A .Yes, they are B .Yes, he is

C. Yes, that is D. Yes, it is

( )14.—Are those maps?

--Yes ,______

A .those are B. they are

C. they’re D. these are

( )15.—Where is my football?

--It’s ___the table ____the floor.

A .on, under B .on ,on

C .under, under D . under, on

四.用所给词的适当形式填空。(10分)

1.There are some _____(watch) over there.

2.My brother ___(play)volleyball ry day..

3.____(she) family name is Bush.

4. Bruce ____ (not eat) oranges.

5. Let ____(we) watch TV.

五.句型转换。(5分)

1.C-A-S-E, case.(划线部分提问)

________________________________

2.They are on the dresser.(变一般疑问句并)

作否定回答)

————————————————————

3.The soccer ball is under the desk. (划线部分提问)

_______________________________

4.This is a tomato. (变复数形式)

____________________________

5.He has two brothers.(一般否定句,并作否定回答)

_______________________

六.补全对话,每空一词。(10分)

Bill: What do you like to eat, Sandra?

Sandra: Well ,I like salad, but I 1 like broccoli.

Bill: And your brother and sister? What do 2 like to 3 ?

Sandra: My 4 likes bananas and oranges, but he 5 like hamburgers.

My 6 likes carrots. She 7 vegetables. We like lots of

8 food.

Bill: And I really (真的) 9 cream’s you like cream, Sandra?

Sandra: Yes, I do.

1._______ 2________3_______4_______5_________

6________7________8_______9_______10_________

七.完型填空。(每空1分,共10分)

Do you know the girl? She is 1 good friend. She is 2 English girl. 3 name is Lucy. 4 twelve. My 5 is Fang fang. 6 a Chinese girl. I’m four.

Lucy 7 a all sports collection. She has 8 tennis racket, three basketballs, and four soccer, but I only he 9 baseball. Lucy and I 10 sports ry day.

1.A.I B. you C .me D. my

2.A.a B. an C. the D./

3.A.His B. he C. Her D. Her’ s

4.A. She B. She’s C. Her D. His

5.A.name B. book C. friend D. bag

6.A.I B. I’m C. She D. She’s

7.A.he B. has

C. doesn’t he D. doesn’t has

8.A.one B. five C. three D. four

9.A.a B. two C. three D. four

10.A.plays B play C. playing D. to play

八.阅读理解(20分)

(一)

如果是你,你想玩电脑还是打网球?

Mila: Let’s play comr s.

Ella: That sounds like fun. Where’s your comr ?

Mila: I don’t he a comr. Do you he a comr?

Ella: No, I don’t. But I he a TV. Let’s watch TV.

Mila: No, that sounds boring. Let’s play tennis. I he a tennis racket.

Ella: Good,. Where is it?

Mila: On the sofa. Do you he a racket?

Ella: No, I don’t. Does your brother he a racket?

Mila; Yes, he does. And he has tennis ball. 第4页

( )1.—Does Mila he a comr?

--___________.

A. She has two comrs. B yes, she does.

C. No, she doesn’t.

( )2.—Does Ella he a TV?

--___________.

A . Yes, she does. B .No, she doesn’t.

C . Yes, she is.

( )3.—Does Ella he a tennis racket?

--__________.

A. Yes, she does. B .She has two tennis rackets.

C. No, she doesn’t.

( )4.—Does Mila he a tennis racket?

--___________.

A. She has two tennis rackets. B. Yes, she does.

C. No, she doesn’t.

( )5.—Does Mila’s brother he a tennis rackets?

--___________.

A. He doesn’t he tennis balls. B. Yes, he does.

B. No, he doesn’t.

(二)

你家有这么多人吗?

Nora is an old woman. She has a big family. There are ten people in her family. Her huand is Dan. They he three children, two daughters and a son. Peter ‘s wife is Al. They’re Mike and Amy’s parents. Linda is Rose and John’s child. Becky and Rose are sister .Peter is Linda’s uncle. Becky is Amy’s aunt. Dan and Nora are children’s grandparents.

( )1.—How many people are there in Nora’s family?

--________.

A. ten B. two C. Three D. A big family

( )2.—Who are children’s grandparents?

--__________.

A. Mike and Amy B Linda and Rose

C. Nora and Dan. D Peter and John

( )3.—Who is Linda’s uncle?

--___________.

A. John B. Peter C. Dan D. Mike

( )4.Becky and Rose are _______.

A. brother B. sister C. uncle D. grandparents

( )5.Al is ______wife.

A. Peter’s B. John’s C. Dan’s D. Mike’s

(三)

Dear friend,

My name is Wang Ying. I’m a Chinese girl. I’m 13. I’m from Qing Dao,.

China. Do you know Qingdao ? It’s very beautiful (美丽). I’m in No 1

Middle School. There are 900 students and 50 teachers in our school. There

Are 45 students in our class, 25 girls and 20 boys. My English teacher is

Miss Zhang. I like English. English is interesting. Her class is fun, too.

My math teacher is Mr. Liu. His class is boring. I don’t like math.

It’s so difficult.

I he a good friend. Her name is Zhao Ling. She is 14.

I want to find a pen friend. Please write to me.

Yours,

Wang Ying

1. I’m a Chinese ____________.

2. Miss Zhang is my _____________.

3. Mr. Liu is my ________.

4. Zhao ling is my __________.

5. I like __________.

6. I didn’t like ____________.

7. English is _____________.

8. Math is __________.

9. English teacher’s class is ___________.

10. Math teacher’s class is _________.

九. 书面表达(10分)

你想参加学校的体育俱乐部吗?写一段话,向教练介绍你自己(如:姓名,年龄,班级,

爱好及家人的一些情况和爱好) (字数50-60字)

就这些了……………………

七年级英语期末试卷及

35. A. way B. time C. month D. river

期末作为一种对学期英语 教学 工作 总结 的形式,是对七年级师生一学期的教学效果进行的检测。这是我整理的 七年级英语 期末试卷,希望你能从中得到感悟!

七年级英语期末试题

(满分:100分 时间:90分钟)

题号 听力部分 笔 试 部 分 总分

I II III IV I II III IV V VI VII

得分

部分 听力(20分)

I. 听 句子 ,选择正确。每个 句子读两遍。(每小题1分,共5分)

( )1. A. B. C.

( )2. A. B. C.

( )3. A. B. C.

( )4. A. B. C.

( )5. A. B. C.

II. 听句子,选择正确答语。每个句子读两遍。(每小题1分,共5分)

( )6. A . I’m in Class One. B. I’m in China. C. I’m in Grade Sn.

( )7. A. It’s Tom. B. It’s Mike’s. C. It’s new.

( )8. A. Sorry, I don’t know him. B. Li Dong is at school. C. Yes, you’re right.

( )9. A. Where is the fish? B. Thank you. C. I don’t think your fish is very n.

( )10. A. Good idea. B. OK, playing guitar is my forite. C. Yes, I’d like to.

III. 听对话及问题,选择正确。每组对话和问题读两遍。(每小题1分,共5分)

( )11. A. New. B. Old. C. N.

( )12. A. Apples. B. Bananas. C. Oranges.

( )13. A. 8723-5691. B. 8732-5691. C. 8271-5693.

( )14. A. In an off. B. She teaches English. C. In a school.

( )15. A. He would like a red jacket. B. He would like a white jacket.

C. He would like a yellow jacket.

IV.听短文,选择正确。短文读两遍。(每小题1分,共5分)

( )16. Jim is ____ years old. A. 14 B. 13 C. 11

( )17. Jim’s father is ____. A. a driver B. a teacher C. a farmer

( )18. Jim and Susan study in ____.

A. the same grade B. the same school C. different schools

( )19. Jim is in ____. A. Grade Sn B. Grade Eight C. Grade Nine

( )20. Jim’s family is from ____. A. China B. England C. the U.S.A.

第二部分 笔试(80分)

I. 单项选择。(每小题1分,共10分)

( ) 1. ---________ is the school trip?

---________ May.

A. When;In B. Where;In C. What;On D. When;On

( ) 2. ---What about ________ football after school?

---Good idea.

A. play B. to play C. playing D. plays

( ) 3. ---Do you like this skirt?

---No, I want a short one. This one is _____ for me.

A. big B. all C. long D. short

( ) 4. ---How much ______ the blue socks?

---_________ three dollars.

A. is; They is B. are; They’re C. is; It’s D. are; It are

( ) 5. ---Is this ________ dictionary?

---No, _____ isn’t _______ dictionary.

A. his; it; me B. your; it; my C. your; this; my D. my; it; my

( ) 6. ---Are Bob’s pens ____ the pencil case? ---Yes, _____ are.

A. on, it B. on, we C. in, they D. in, it

( ) 7. ---What’s your forite ? ---Pears.

A. color B. fruit C. day D. vegetable

( ) 8. ---Can you me my math homework, Miss Zhang?

A. with; in B. do; at C. to do; on D. to do; in

( ) 9. I am years old today. And my parents he a party for my birthday.

A. twelfth; twelve B. twelfth; twelfth C. twelve; t welfth D. twelve; twelve

( ) 10. --- , is Jim American? --- , I don’t know.

A. Excuse me, Excuse me B. Excuse me, Sorry

C. Sorry, Sorry D. Sorry, Excuse me

II. 阅读理解。(每小题1分,共20分)

(A)

There is a little cat in the house. She likes playing with a ball. The ball is very n. It is the cat’ s good friend.

One day the little cat makes a mistake. The owner ties (系) her to a big tree. She wants to play with the ball. But she can’t get it. The little cat is very sad. “How can I get the ball?” the little cat says. She uses her forepaw (前爪) to get the ball. But it is too hard. She can’ t get the ball.

The little cat thinks and thinks. Then she has an idea. She turns round (转身) and soon she gets the ball. Now she can play with the ball again. How happy she is!

But do you know how she gets the ball?

根据短文内容判断正(T)误(F)

( ) 1 .The ball is the cat’s good friend.

( ) 2. The owner ties the cat to a big tree, because she doesn’t catch the mouse.

( ) 3. She can get the ball easily(轻易地).

( ) 4. The Chinese for the word “hard” is “困难的”.

( ) 5. The cat is very happy because she can get the ball at last ().

(B)

All around the world, people drink tea. But tea does not mean the same thing to ryone. In different countries people he very different ideas about drinking tea.

In China, for example, tea is always served when people get toger. The Chinese drink it at any time of the day at homes or in teahouses. They prefer their tea plain, with nothing else in it.

Tea is also important in Japan. The Japanese he a special way of serving tea called a tea ceremony(仪式). It is very old and full of meaning. Everything must be done in a special way in the ceremony. There is n a special room for it in Japanese homes.

Another tea-drinking country is England. In England, the late afternoon is “teatime”. Almost ryone has a cup of tea then. The English usually make tea in a teapot and drink it with milk and sugar. They also eat cakes, cookies and little sandwiches at teatime.

In the United States people drink tea mostly for breakfast or after meals. Americans usually use tea bags to make their tea. Tea bags are faster and easier than tea in teapots. In summer, many Americans drink cold tea — “d tea”. Sometimes they drink d tea from cans, like soda.

阅读短文,选择正确。

( ) 6. Iced tea is popular _______.

A. in winter B. for breakfast C. in England D. in the USA

( ) 7. The English like to _______.

A. drink their tea plain B. eat cakes and cookies with their tea

C. he tea with dinner D. drink their tea in a special room

( ) 8. The Chinese drink tea _______.

A. in a special ceremony B. only in teahouses

C. when they get toger D. for breakfast

( ) 9. Tea is popular _______.

C. only in the USA D. all around the world

( ) 10. The passage is about _______.

A. Chinese tea B. different ways of drin king tea

C. the teatime in England D. why tea is important

(C)

My ideal school is not far from my home. It takes me 15 minutes to walk there. In the school, there are a lot of trees and flowers. It’s very beautiful. We also he a big school library, a swimming pool and sn comr rooms. We begin our classes at 8:00 a.m. and finish them at 4:00 p.m. So we he a lot of time for after-school activities. Every day we only do our homework for an hour. Every month we can go on a school trip. How wonderful the school life is!

阅读以上短文,完成下面各题,每空一词。

11. The boy goes to his ideal school on ________.

12. There is a ________, a swimming pool and sn comr rooms in his school.

13. His school ________ at 4:00 p.m.

14. The students he ________ time for after-school activities.

15. They often go on a school trip ry ________.

(D)

Riding a bike is good exercise and great fun. But what do you do with your old bikes? American girl Nicole Basil has a wonderful answer to this question. She sets up Pedal Power — a charity (慈善团体) to collect (收集) old bikes and gives them to people wh o need them.

Nicole, 12, starts Pedal Power when she is only 8. She collects more than bikes each year. Every November, Pedal Power holds a bike race t o ask for more donations (捐赠品).

Nicole also gives bikes to students as prizes. Top students and hardworking students both get them. “Some kids aren’t as lucky as others, but they still do well in school,” she says. “And I think they should get a prize.”

A bicycle shop s Nicole check (检查) ry bike for the races. The owner says he is happy to the charity. “This is just wonderful,” he says. “We need to learn from Nicole to others.”

阅读短文,回答下列问题。

16. What does Nicole do with the old bikes?

_________________________________________

17. How long has Nicole worked at Pedal Power?

________________________ _________________

18. When does Nicole hold a bike race to ask for more old bikes?

_________________________________________

19. What kind of students can get the bikes as prizes?

_________________________________________

20. What does the shop owner do to the charity?

_________________________________________

III. 补全对话。(每小题2分,共10分)

在空格处填入合适的句子完成对话,使对话完 整并正确。

Jenny: Hi, Paul! Lets’ play comr s!

Paul: (1)_______________________________, Jenny, but I don’t he a comr.

Jenny: Well, do you he a volleyball?

Paul: (2)_______________.

Jenny: Then let’s play volleyball.

Paul: Oh, (3)___________________________

Jenny: OK, let’s watch TV.

Paul: That sounds boring. Hmm…(4)__________________________ Do you he a soccer ball?

Jenny: No, I don’t.

Paul: Oh. Well, (5)_____________________________

Jenny: Yes, I do. Let’s play basketball!

Paul: That sounds fun!

IV. 完成短文。根据短文内容填入所缺单词,每空一词。(每空1分,共10分)

(A)

Do You He a Soccer Ball?

Frank Brown:

I don’t he a soccer ball, but my brother Alan (1)______. We go to the same school and we love soccer. We play it at school (2)_______ our friends. It’s relaxing.

Gina Smith:

Yes, I do. I he two soccer balls, three volleyballs, four basketballs and five baseballs and bats. I love (3)_______, but I don’t play them ---I only watch them on TV.

Wang Wei:

No, I don’t. Soccer is (4)___________. I like ping-pong. It’s easy for me. I he three ping-pong balls and two ping-pong bats. (5)________ class, I play ping-pong with my clasates.

(B)

Come and buy your clothes at our great sale! We sell all our clothes (6)_______ very good prs. Do you like sweaters? We he green sweaters (7)________ only $15! Yellow sweaters are only $12! Do you need (8)__________? For boys, we he black trousers for only $22. and shorts are only $16! For girls, we he skirts in purple for only $20. How much (9)______ our jackets? Only $30! And we he black shoes for only $28. Socks are only $2 for three (10)________!

V. 选词填空。(每空1分,共10分)

从方框里选择合适的单词,并用其适当形式完成下列短文。(有两个是多余的)

an, of, dress, England, grade, it, that, arm, say, know, shirt, how

Hi, my name is Tony. I’m _____(1 ) English boy. I am from ________ (2). I’m a high school student ________ (3) Hefei No.1 High School. I’m in Class Two, ________ (4) Sn. My father and mother work in Hefei now. Look, my father is in a white ________ (5), he is tall and he has a long face. The woman in a yellow ________ (6) is my mother, her eyes are big and blue. The baby in my mother’s ________ (7) is my little sister, she is only two years old. We all like her very much. She has a n doll, ________ (8) name is Barbie (芭比娃娃).

Who is the boy in the picture? Do you _______ (9)? It’s me. My friend _______ (10) I look like my father. Do you think so?

1________ _ 2_________ 3_________ 4____________ 5_____________

6_________ 7_________ 8_________ 9____________ 10_____________

VI. 翻译句子。(每句2分,共10分)

1. 汤姆是个很有趣的人,我们都喜欢和他在一起。

Tom’s good fun. We all enjoy ____________________________.

2. A: 王老师上课非常有趣。 A: Mr. Wang’s English class is very interesting.

B: 的确如此。 B: ___________________________.

3. 请来和我们吃顿饭吧。 Please _________________________________.

4. 这双鞋穿在你的脚上真好看。These shoes __________________________________.

5. 历史和语文是我最喜欢的科目。 ____________________________________________.

VII. 书面表达。(共10分)

下面图表里是Lily和Jack两个人的资料, 请根据图表所列的信息, 写一篇小短文, 介绍两人的情况。要求60-70词。开头已给出。

Name Country Fo rite color/food/animals Get up Go to bed

Lily England pink, jiaozi, pandas 6:30 9:30

Jack Canada blue, hamburgers, lions 6:40 9:50

Lily is an English girl. She likes pink very much. __________________________________

_____________________________________________________________________

七年级英语期末试卷参

部分 听力(20分)

(每小题1分,共20分)

I. 1-5 CACBC

II. 6-10 CBABA

III. 11-15 ABBCA

IV. 16-20 CABAB

第二部分 笔试(80分)

I. 单项选择(每小题1分,共10分)

1-5 ACCBB 6-10 CBCCB

II. 阅读理解(每小题1分,共20分)

1-5 TFFTT 6-10 DBCDB

11. foot 12. library 13. finishes/ends 14. much 15. month

16. She collects them and gives them to people who need them. 17. About four years. 18. Every November. 19. Top students and hardworking students. 20. He s check ry bike for the races.

III. 补全对话(每小题2分,共10分)

1. That sounds interesting/n/Good idea… 2. Yes, I do. 3. I don’t like it./ it’s boring. /volleyball is so difficult…. 4. Let’s play soccer!/I like playing soccer….. 5. do you he a basketball?

IV. 完成短文(每小题1分,共10分)

1. does 2. with 3. sports/them 4. difficult/hard 5. After

6. at 7. for 8. trousers 9. are 10. pairs

V. 选词填空(每小题1分,共10分)

1. an 2. England 3. of 4. Grade 5.---OK. Come to my house Sunday morning. shirt 6. dress

7. arm(s) 8. its 9. know 10. says

VI. 翻译句子(每小题2分,共10分)

1. being/staying with him 2. That’s for sure. 3. come and he dinner with us

4. look n on you 5. History and Chinese are my forite subjects.

VII. 书面表达(共10分)

参考 范文 :

Lily is an English girl. She likes pink very much. Jiaozi is her forite food. Her forite animals are pandas. In the morning, she gets up at 6:30 and in the ning she often goes to bed at 9:30. Jack is from Canada. His forite color is blue. He says hamburgers are his forite food. He likes lions very much. He often gets up at 6:40 in the morning and goes to bed at 9:50 in the ning. Lily and Jack are both in Class One. They are good friends.

七年级英语期末试卷及相关 文章 :

1. 七年级上册英语期末试卷及

2. 七年级英语下册期末试卷及

3. 七年级英语上册期末六校联卷及

4. 七年级英语上册期末试题

5. 七年级英语上册期末题

人教版七年级上册英语期末测试卷

My English teacher, Miss Smith is 26 exciting teacher. Everyone likes 27 very much. She works very hard, 28 she is often very tired. Her home is about 10 kilometers 29 school. She gets up 30 six o’clock ry day,and has a quick 31 .Then she 32 school.First,she walks to the bus station. It takes about ten minutes. 33 the early bus takes her to school. The bus usually takes about 25 minutes. How long does it 34 her to school? Do you know? 35 Minutes.

英语期末就要到了,为让七年级同学们对期末有更好的准备,下面是我为大家精心整理的,仅供参考。

人教版七年级上册英语期末测试题

一、单项选择10分

11.I am good at pictures. I‘d like an art teacher.

A draw, to be B ,draw ,being C drawing ,to be D drawing ,being

12.There is desk in the room. old book is on it.

A a, A B a, An C ,An,a D an,A

13. your father usually e home after work?.....At about 6:30.

A How is B How does C When does D When is

14.My mother usually at 6:00a.m in the morning.

A gets up B get up C get to D gets to

15.My parents to go out at night.

A not want B not wants Cdoesn’t want D don’t want

16…. do you play r s?.... Once a week

A How B How often C How old D How many

17.Tom goes to school by bus because his school is very close to his home.

A always B seldom C often D sometimes

18.My brother enjoys basketball and he loves guitar,too.

A playing,playing the B playing,play the

C to play,playing D playing the,playing

19… your sister sports ……..No,she

A Do,like,don’t B Does,likes,doesn’t

C Does,like,doesn’t D Do,like,do

20. he doesn’t he breakfast, he feels gry.

A Because,so B/,so C /,because D So,/

二、语法选择10分

26. A a B the C an D/

27. A she B her C hers D him

28. A and B but C so D for

29. A on B in C for D from

30. A at B on C in D of

31. A lunch B breakfast C supper D dinner

32. A goes B go C goes to D go to

33. A First B Then C Last D Than

34. A take B takes C taking D is taking

35. A twenty-five B Ten C Thirty-five D Forty-five

三3. 我家附近一间书店。、完形填空10分

Jim es from America. Now he studies in No.5 Middle School in Beijingry morning he 36 No. 21 bus to school .He has four classes in the morning and two in the 37 .

He speaks English very well .He often s his Chinese friends learn 38 .He doesn’t like maths. He think it’s boring. 39 forite subject is P.E.He thinks it’s 40 . His P.E 41 is very strict with him. His last class ry day 42 at 4:00. He can be in all kinds of activities in the 43 .Jim wants to know more about Chinese culture. From Monday to Wednesday he often does Chinese Kung fu.On Thursday he often play chess for 44 hour. OnFriday he takes part in the activities of 45 club. He likes playing guzheng and Erhu .Every afternoon he gets home about six’o clock. How busy his day is!

36.A takes B has C gets D he

37. A noon B night C afternoon D ning

38. A Chinese B English C Japanese D American

39. A He B His C She D Her

40. A boring B bad C funny D relaxing

41. A friend B mother C teacher D brother

42. A ends B begins C works D starts

43. A night B ning C morning D afternoon

44. A the B an C a D/

45. A music B painting C chess D basketball

四.阅读理解20分

My name is Rechel.I am a student in Toronto,Canada.It is the second largest country in size in the world.Every day my brother Andrew and I go to school at 9:00.i dislike getting up at 8:00 in the morning! When we finally get up,weneed to eat breakfast quickly at home and walk to school.At school we learn English and French,because Canadian speak English andFrench. Of course,we also learn history,art,science and math.My brother’s forite sport is football. ButI can’t play football at school—there are no grassy area on the playground.School ends at 3:30 Andrew and I walk back home. I he to do a lot of homework first. After dinner I can relax. My forite hobby is going for a boat-ride with dad. Sometimes go to the park. I hope you can enjoy learning about my life.

1. is the second largest in the world.

A Canada‘s size B Canada’s population C the number of the students

2.Rachel and his brother go to school .

A in the school bus B in their father’s car C on foot

3.what language does Rachel learn at school.

A English and science B English and French C French and reading

4.Rachel’s brother likes best.

A volleyball B football C baseball

5.Rachel has to first after school.

A his mother cook B do his home work C watch TV with his brother

Hello I am Jeff Smith. I work in a big r pany. I am busy ry day. I want to rent an old house with a beautiful garden. The house must be big and clean. And it’s quiet in the neighborhood .I hole there is a beautiful park nearby. So my parents can take a walk in the morning or after supper .Our family can he fun there on weekends,there must be a school near the house .and mydaughter can go to school by herself .she likes playing r s and wants a video arcade 中心 near the house .my wife is busy with her work .She hopes there is a big supermarket in the neighborhood .she can buy some food and drinks easily. Do you he such a house to rent?Please call me at 13827317567.

1.There are_____people in Mr. Smith’s family.

A.3 B.4 C.5 D.6

2.Jeff wants to rent _____.

A. a garden B. a new house C. a park D. a house with a garden

3.Why does Jeff hope these is a big park in the neighborhood?

A. His parents can take a walk in the morning or after supper.

B. His family can he fun there on weekends.

C. A and B.

D. His father can work there.

4._____ likes playing r s.

A. Jeff’s wife B. Jeff’s daughter C. Jeff D. Jeff and Jeff’s wife

5.This is an ______ .

A. ad.广告 B. letter C. story D.

Jim’s Timetable课程表

Time Period Monday Tuesday Wednesday Thursday Friday

8:00-8:45 1 Chinese Math English Chinese English

8:55-9:40 2 English Chinese Math English Math

10:00-10:45 3 History English Biology Math Chinese

10:55-11:40 4 Math P . E. Politics P .E. Geography

14:30-15:15 5 Biology Music Chinese history Art

15:25-16:10 6 Geography Art Biology r Class meeting

1. Jim has _____English lessons ry week.

A. 5 B. 6 C. 7 D. 8

2. P .E. class begins at _____ on Tuesday .

A. 10:00 B. 10:55 C. 8:00 D. 14:30

3. Jim is hing a an ____ lesson at 15:00 on Monday .

A. music B. geography C. biology D. art

4. Class meeting is on _____ .

A. Wednesday B. Friday C. Thursday D. Tuesday

5. Jim has a r lesson on _____ .

A. Tuesday B. Wednesday C. Thursday D. Friday

Many children like to watch TV . It’s very interesting. The programs on TV l us many things about the country and a lot of news about the world, Judy watches TV on Sundays. There is an English program getting to know America. It’s her forite program.It ls about the life of America, such as buildings,traffic, food, houses, movie stars. Why does she love the program? Because she likes English. She likes English best of all the subjects. When she is watching the program, she can remember ry word she sees on TV. That s a lot, because she learns many new words, English really interests her令她感兴趣. She likes learning English. On weekdays, she doesn’t watch TV, but reads English story books ry night before she goes to bed. She can learn a lot from the TV program and the story books.

1. When does Judy often watch TV?

A. On Sundays. B. On weekdays. C. At nigh. D. On Saturdays.

2.Getting to know America is aan______program.

A. American B. English C. Japanese D. Chinese

3.Why does she love the program?

A. She likes English. B. She can learn a lot from it.

C. It s her a lot. D. A, B and C.

4. The program is about_____.

A. English stories B. American food

C. the life of America D. American people

5. Which one is true ?

A. Judy learns English at school and home.

B. Judy watches her forite TV program on Sunday.

C. Judy reads English books ry day.

D. Judy likes English a little.

五、首字母填空10

1. Everyone in my class likes Jack because he is very f______.

2. Tell me about your h______.----Well, I like reading very much.

3. The party usually starts at 8 p.m,and e______ at 11 p.m.

4. Ben always r______ a bicycle to school .

5. You can watch TV after you f______ your homework.

6. He is a good student, and he is n______ later for school.

7. The bell r______, Let’s go to he a class.

8. My sister’s d______ is to be an engineer and she works hard for it.

9. If you want to win the match, you need a lot of p______ first.

10. Mum goes to the m______ to buy vegetables and fruit.

六、完成句子14分

1. 周末我们通常过得很愉快。

We usually ______ ______ ______ ______ on weekends.

2. Sally 经常在课后参加篮球比赛。

Sally often ______ ______ ______ basketball matches after school.

There is a bookshop ______ ______ my home.

4. 没有人想与吉姆交朋友。

Nobody wants ______ ______ ______ ______ Jim.

5.我愿意帮你学英语。

I ______ ______ ______ you with your English.

6.我每天刷牙两次。

I ______ my ______ tw a day.

7.你每天晚上10点钟睡觉吗?

Do you ______ ______ ______ at 10:00 ry day?

七、书面表达16分

假设你是初一2班的学生,请根据表格中的提示,以My daily life为题写一篇80字左右的文章,介绍你的日常生活。

上午 6:30 起床,然后刷牙、洗脸,吃早餐的时候喜欢听英语。

7:10步行去上学。

8:00 上节课。每天上8节课,上午5节,下午3节。

下午 中午通常在学校吃午餐。4:50放学后参加体育锻炼40分钟。和同学一起打乒乓球。

晚上 7:30 吃晚饭,写一个半小时作业。然后上床睡觉。

一、单项选择

:11-20CBCAD BBABB

二、语法选择

CACDA BCBAC

三、完形填空

ACBBC CADBA

四.阅读理解

ACBBB CDCBA ABCBC ABACD

五、首字母填空

1、funny 2、hobby 3、ends 4、rides 5、finish 6、nr 7、ringing 8、daughter 9、pract 10、market

六、完成句子

1、 he a good time 2、 takes part in 3、close to 4 、to make friends with 5、would like to 6、brush te 7、go to bed

七年级英语综合测试题(2)

VI

BWhen the world was very young, people lived only in hot places. They did not live in cold places because they could not keep warm there.

At first men did not know how to make a fire. Sometimes trees were hit by lightning. Then a fire was started. People took some of the fire near their homes. A fire was very important for three reasons. It kept them warm. Wild animals were frightened when they saw it. And when food was cooked, it tasted much better.

But men did not know how to make a fire. When they had a fire, they did not let it stop burning. If it went out(熄灭), they could not start it again. They had to wait for lightning to start another fire. Sometimes they had to wait for years.

Later, they learned how to make a fire. But it was not easy to make soming burn.

Now we he matches. We can carry them in our pockets and make a fire when we want to.

5. People found that .

A. the cooked food tasted better than the uncooked food

B. the uncooked food tasted much better than the cooked food

C. wild animals enjoyed the cooked food

D. wild animals kept warm near the trees

6. People didn’t let a fire stop burning because .

A. they were not able to make a fire B. they liked to wait for lightning

C. it rained now and then D. they didn’t like to make a fire

7. Now we can make a fire .

A. by lightning B. if it is necessary C. for three reasons D. in our pockets

CElephants are interesting animals with large ears, a long trunk, and short legs. They use their legs to stand and walk. Unlike lions and tigers, an elephant cannot use its legs to jump or run fast. But elephants do he soming that most animals do not he —a trunk!

An elephant uses its trunk as a monkey might use its arms and hands. The elephant can scratch(抓)an itch(痒), pick up a all peanut or a piece of popcorn, and touch objects with its trunk. An elephant’s trunk can n wrap(裹)around a tree and pull it out of the ground.

An elephant’s trunk also s it drink water. An elephant can drink as much as 50 gallons(加仑) of water into its trunk. Then the elephant squirts(喷出)the water into its mouth and down its throat. An elephant’s trunk is very useful!

8. In paragraph 2, less means .

A. without B. nr C. trying to D. full of

9. What is one way that the elephant’s trunk is like a monkey’s hands?

A. Both he fingers. B. Both squirt water.

C. Both hold food. D. Both he fingernails(指甲).

10. This story mostly ls the reader that elephants .

A. use their short legs to find food B. are stronger than most animals

C. drink water once a day D. would be less without a trunk

四、根据所给中文或单词写出正确的形式:(每题1分,共15分)

1. He (重)about four kilograms when he was born.

2. Every year (千)of foreign visitors come to Suzhou.

3. He is very young, so he can’t look after (自己).

4. Chen Dan is a clr young boy. He can learn things (快).

5. Do you know that he is fond of (收集)stamps?

6. There are lots of (老鼠)in the old man’s house.

7. Sandy’s sister (刷)her cat’s fur once a day.

8.He wants to be a (科学家)when he grows up.

9. I wanted to see the rainbow after the rain. But it didn’t (出现).

10. When I woke up, I heard somebody (敲)on my door.

11. The students of Class 8 are planning (visit) Guilin this summer.

12. I look forward to (own) a comr.

13. Julie (feed) her pet last night? Yes.

14. Shall we go (row)tomorrow? A good idea.

15. Mr. Wang (find) a job in a big city two years ago.

五、汉译英:(每小题3分,共15分)

1、桑迪能教我弹钢琴。

2、乘宇宙飞船从地球到月球大约要3天时间

3、他冲进厨房,帮助他的邻居扑灭了火。

4、你害怕独自在家吗?不,我一点也不怕。

5、昨天一个8岁的女孩在上学的路上摔倒了。

六、书面表达:(共15分)

学生会要表彰一批学生,Andy想他的同学Peter竞争最乐于助人奖(Most Helpful Student Award)。请以Andy的名义给学生会主席写一封信。词数约80,不包括已给出的开头和结尾。

信中必须包括下列要点:

1、乐于助人,有礼貌,勤奋学习,常帮助同学学习

2、经常为希望工程募集物品,照顾老年之家的老人

3、电脑用得好,为帮扶俱乐部(the Helping Hands Club)从网上获取资料

4、上周一看到一个小男孩迷路,将他送回家,他的父母很感激他。Andy回到家时,已经很晚了,但他感到很开心。

Dear Sir

I would like

We will be glad if he can win the award.

Thank you

Yours faithfully

Andy

【试题】

一、听力选择(共24分)

题号12345678910

BBBBACABCA

题号111213141516171819

BCABAABAB

二、选择填空:(每题1分,共31分)

题号1234567891011

ACBDCCDCBDA

题号1213141516171819202122

DCDDDBCDCAB

题号232425262728293031

CBABCBBCA

三、阅读理解:(每小题2分,共20分)

题号12345678910

CBDCAABACD

四、根据所给中文或单词写出正确的形式:(每小题1分,共15分)

1. weighed 2. thousands 3. himself 4. quickly 5. collecting

6. m 7. brushes 8. scientist 9. appear 10. knocking/knock

11. to visit 12. owning 13. Did, feed 14. rowing 15. found

五、汉译英:(每小题3分,共15分)

1. Sandy can teach me to play the piano.

2. It takes about three days to trel from Earth to the moon by spaceship.

3. He rushed into the kitchen and ed/to his neighbour put out the fire.

4. Are you afraid of staying at home alone? No, I’m not afraid at all.

5. Yesterday, an 8-year-old girl fell down on her way to school.

六、书面表达:(略)(共15分)

听力录音材料

A. 回答问题:听下面6个问题。每个问题后有一个小题,从题中所给的A、B、C三个选项中选出选项。每个问题读两遍。

1. What’s your fourite pet?

2. Would you like to go to the cinema with us?

3. What’s your friend like?

4. Who does the dog belong to?

5. How often were you late for work last year?

6. What are you going to buy for your sister?

B. 对话理解:听下面8段对话。每段对话后有一小题,从题中所给的A、B、C三个选项中选出。每段对话和问题读两遍。

7. W: Would you like to go fishing with us this Sunday, Jack?

M: Yes, if it is fine.

8. W: I know Paris well. I lived there for five years before I moved to China.

M: In China, which city do you live in?

W: Shanghai.

9. W: Did you watch TV last night, Bill?

M: Yes, and because I watched TV I didn’t finish my homework.

10. W: The film started? When did it begin?

M: Ten minutes ago . It will be over in an hour.

11. W: I want to he a little dog as a pet.

M: We he one here. It is very lovely. You will love it.

12. W: What time i3. A. He likes basketball. B. He's tall with cy hair. C. He is a doctor.s it?

M: It’s 7 in the morning.

W: Did you feed your fish?

M: Yes, I fed it 20 minutes ago.

13. W: Excuse me, how can I get to the bookshop?

M: Go along this street to the end. You can’t miss it.

14. W: What subject do you like best, Simon?

M: I like Maths best. What about you, Suzy?

W: I like Maths, too. I also like History and Chinese. But my fourite subject is English. I he good grades in it.

C、短文理解:听下面的短文。短文后有5个小题,从题中所给的A、B、C三个选项中选出。短文读两遍。

We look at the moon ry night. It’s bright and beautiful. We write songs and poems about it.

In the 1960s, some countries had a dream to be the first country to put a man on the moon. America and Russia were part of this ‘space race’. They spent a lot of time and money on it. The first man in space was Yuri Gagarin, a Russian. That was in 1961. Years later, in 1969, an American spaceship flew to the moon. Neil Armstrong walked out. He was the first man to walk on the moon. The race to moon was over.

China entered the space race in 2003. Yang Liwei, China’s , flew around the Earth in that year.

只要我们大家能够在我们的期末复习之前,通过我们的初一英语试题,进行我们英语知识系统的复习,大家的英语成绩就一定不会让大家失望。

版权声明:图片、内容均来源于互联网 如有侵权联系k19126499204@163.com 删除